You are on page 1of 84

'1 IlFRIVADA PARCIAL DE UNA FUNCiÓN 1>1<: V tUA, \' ARIABLES

1 11 /II/Iicos de Cálculo Vol. f , hemos visto quc para l'stlldi,lr 1,1 pendlcllIe de la
I ,1,1 langente a la gráfica de una función [ en un punlo y P,II ,1 Itl..:dlr 1.\ 1"pidl" de
IIlIbio dI.! la variable dependiente respecto a la vallahll' ilulcIWlldil'llk' l'I,1
11 u'\,lrio emplear la derivada de la función [. En este I.apitlllo \l'I\;11l0~ conlo

, IS idl:as se generalizan a funciones de varias variables.

Udlllid6n 1.- Sea [: D ~ IR$.2 ...... IR$. una función de dos variables con dOll\illio CII
1, Ol1ll1l1to D ~ IR$.2
1\, dl'llvadas parciales de primer orden de [ con respecto a las varialJks
111 l 'puHlientes x e y, en cualquier punto (x; y) E D, son las funciones dadas por
a[(x' y) ~" [(x + h' y) - [(x' y)
a ' = [x(x; y) ';;"DJ(x ; y) = lim ' h '
x ~o. .

éJ[(x; y) = [y(x; y) = Dd(x; y) = lim [(x + h; y~ - f(x; y)


ay k-->O

I" lo~ Ifmites existen.


fl,~

1 m 1IIIIIII.!s en esta definición son en una variable, por lo cual para calcularlos se
JI 11 , dl IIsar las técnicas aprendidas en Tópicos de Cálculo Vol. /, sobre
11l10l1alización, regla de L'Hospital, etc.
1'11 '\10 que la derivada parcial de una función de dos variables es la derivada
1IIII¡IIana de la función que se obtiene al fijar constante una de las variables x e y,
.11 lórlculo se realiza de la misma manera y usando las mismas reglas que se
111 rll/"l1 para las funciones de una variable real.

e )lI\l'l'\ ación 1.- Cuando queremos definir la derivada de [en un punto particular
( \ ,,: Vil) E f) , simplemente reemplazamos (x; y) por (xo; Yo) en la definición.

I l' \ll1I1() 1. Dada la función [(x; y) = 3x 2 y + X + Y . Usando la definición de


" 11\;¡d;¡ parcial calcule [xCI; 1) Y !y(- I; 1).
Solución
. f(l + h; 1) - f(l; 1) . [3(1 + h)Z + h + 2] - S
fx(l; 1) = h40
hm h = h40
hm h

h(3h + 7)
= h40
Iim
h
=7
. f(-l;l+k)-f(-l;l) . [3(1+k)+k]-3
f y(-l;l)=hm
k40
k =hm k
k40

4k
= lim-
k ... O k
=4
Definición 2.- Sea f: D ~ IR!.n --t IR!. una función de n variables con dominio el
conjunto D ~ IR!.n, tal que z = f(x l ; XZ; ... ; x n )

Las derivadas parciales de primer orden de f con respecto a las variables


Xl; Xz; ... ; X n en cualquier punto (Xl; Xz; ... ; X n ) E D son las funciones de n
variables dadas por:
:~-'
8f(x l ; ... ;xn ) -". f(x l ; ... Xi + h; ... ;xn ) - f(x l ; .. . ;xn )
---:---- =
8Xi
Ix.L(Xl; ... ; X n ) = ¡HO
hm h

si estos límites existen para i = 1,2, ... , n.

La derivada parcial de una función de n vari~s es su derivada de f respecto a


una de sus variables independientes y mantiene ·átJas otras como constantes.

Ejemplo 2.- Halle las derivadas parciales de primer orden de las siguientes
funciones

a) f(x;y) =x3 - 2x 2 yZ + 3 b) 9(X;y) = e x2 - y2 + In(x Z + yZ - 4)

e) h(x; y; z) = 2 cos(xyZ) + tan(yz) -ln(x Z - 4y) + Jxyz

d) fex; y; z) = fZ et~,dt + IX cos(t 2 )dt + arctan(xyz) + 8


x -y

Solución
a) Al derivar f con respecto a X manteniendo constante y, se tiene
fAx; y) = 3x 2 - 4xy 2
Al derivar f con respecto a y manteniendo constante x, se obtiene
fy(x; y) = -4x Zy
'" I derivar 9 con respecto a x manteniendo constante y, resulta
2 2 2x 2 2 2x
1/, (x; y) = eX -y . 2x + = 2xe x -y + ---,--....,--
• XZ + y2 - 4 XZ + y2 - 4
¡\ 1 derivar9 con respecto a y manteniendo contante x, se obtiene
2 2 2y 2y
1/ ,(x; y) = eX -y (-2y) +
> X2 + y2 - 4
= -2ye X2 -y 2 + --;:---::-....,-
X2 + yZ - 4

l' -\1 derivar h con respecto a x manteniendo constante y y z, resulta


2x yz
hx(x; y; z) = -2 sen Cxy2)(y2) - +---==
. X2 - 4y 2Jxyz

= -2 y z sen (xyZ) -
2x JYi
+--
XZ - 4y 2VX
Las derivadas parciales de h con respecto a y y z son:
-4 xz
Il y (X;y;z)=-2sen(xy 2)(2xy)+sec 2 (yz)(z)- 2 4 + r:;;::;;;
x - Y 2...¡xyz

= -9xy sen.Jxy2) + z sec 2(yz) + 2


4
+
rxzr::;
~ x - 4y 2,,¡y

h z (x; y; z) = sec 2(yz) (y) +


xy
r:;;::;;; = y sec 2(yz) +
.¡;yr::
2,,¡xyz 2vz
u) Las derivadas parciales de f con respe~ x, y y z son:
'1>.
2 0yz
fAx; y; z) = -eX + z cos(x 2 ) + 1 +x 2 YZ z Z
xz
fy(X; y; z) = z cos(yZ) + Z Z Z
1 +x Y z

fz(x; y; z) = e Z
2
+ IX cos(t 2 ) dt + 1 xy
Z ZZ
_y +x y z

XY(XZ _ yZ)
si (x; y) :¡: (O; O)
Ejemplo 3.- Sea f(x;y) = xOZ + yZ '
{
si (x; y) = (O; O)
Ilalle fAO; O) y fy(O; O) si es que existe.
Solución
i) Para y :t= O, se tiene
hy(h2 _ y 2)
- O
--"-,;>,;--,---....;,-~
. feO + h;y) - f(O;y) . h Z + y2
fx(O; y) = hm -- = hm - - - . : ,-
h-+Q h h-+Q h
· hy(h 2 - y2) . y(h2 _ y2)
= hm = hm = -y
h->O h(h 2 + y2) h->O h2 + y2
Luego, para y = O resulta [x(O; O) = O

ii) Para x "* O, se tiene


xh(x 2 - h 2)
. [(x; 0+ h) - [(x; O) . X2 + h 2 - O
[, ( x; O) = h m = h->O
h m ----'-'--"""'---';-'----
y h->O h h

Luego, para x = O se obtiene [y(O; O) = O

INTERPRETACIÓN GEOMÉTRICA DE LAS DERIVADAS PARCIALES


DE UNA FUNCIÓN DE DOS VARIABLES

Sea z = [(x; y) una función''4e dos variables con dominio D ~ 1R2 tal que
[x (x o; Yo) Y [y (x o; Yo) e;-..isten para; (xo; Yo) E D.
Si Y = Yo (plano paralelo al plano XZ), entonces z el: = [(x; Yo) representa la
curva formada por la intersección de la superficie z = [(x; y) con el plano
Y = Yo, como se muestra en la figura 3. 1.

'kr~I P,I
I ~
I
• ,
: = f(xy) == ) l .r· ;v ,

y y

Fig.3.1

representa la pendiente de la recta tangente (L r ) a lá curva el en el punto


Po (xo; yo;f(xo; Yo)). (Fig. 3.1)
I l'cuación cartesiana de la recta tangente Lr en el punto Po (xo; Yo; f(xo; Yo)) es
Lr: Z - Zo = fx(xo; YO) (x - xo) /\ y = Yo (zo = f(xo; Yo))

I iI forma vectorial de la ecuación de ia recta tangente Lr es

y su forma paramétrica

x = Xo + t
Lr: y = Yo , t E IR
fZ = Zo + t fAxo; Yo)
donde su vector dirección es a = (1; O; fAxo; Yo))
De forma similar,

F ( • ) _ l' f(xo; Yo + k) - f(x o; Yo)


Jy xo,Yo - 1m
k--+O
k

1L'presenta la pendiente de la recta tangente (L' r) a la curva e 2 (obtenida por la


Intersecc ión de la superficie Z = fex; y) con el plano x = xo) en el punt~
J'n(x o; Yo; f(x o; Yo))·
La ecuac ión cartesiana de esta recta tangente L' s

L' r: Z - Zo = fy(xo; Yo)(y - Yo) /\ x =<~o


, y - Yo Z - Zo
~ L r: - - = JyF ( x ) /\ x = Xo
1 o; Yo
La forma vectorial de la ecuación de L' T es

y su forma paramétrica es

L'r:¡;:;~+s ,sEllt
Z = Zo + s fy(x o; Yo)
donde b= (O; 1; fy (x o; Yo)) es su vector dirección.

Ohservación 2.- Los valores de fx(x; y) y fy(x; y) en d punto Po(xo; Yo; zo) de
1.1 superficie Z = f(x; y) denotan la pendiente de la superficie en las direcciones
de los ejes X e Y respectivamente.
PLANO TANGENTE Y RECTA NORMAL A UNA SUPERFICIE

Definición 3.- La ecuación general del plano tangente a la superficie


z := f(x; y) en el punto Po (xo; Yo; zo) (zo := f(x o); Yo)) con vector normal

1 J k
Ñ:=axb:= 1 O fx(xo;Yo) :=(-fxC x o;Yo);-fy (xo;Yo);l)
O 1 fy(xo; Yo)

:= -(fx(xo; Yo);fy(x o; Yo); -1)


es

Definición 4.- La recta normal a la superficie z:= f(x; y) en el punto


Po (xo; Yo; zo) es la recta que tiene la dirección del vector normal del plano
tangente a la superficie en Po; su ecuación vectorial es

Ejemplo 4.- Halle la ecuación vectoría"1 de la recta tangente a la curva de


intersección de la superficie z = f(x;y) := ~64 - 5x2 -7y2 y el plano x = -2,
en el punto Po( -2; 2; 4).
Solución
Al deri var f con respecto a Y manteniendo x cO-lj),stante (x := -2), se obtiene
-14y 7y
fy ex; y) = --¡:=======;:
2~64 - 5x2 - 7y2 ~64 - 5x2 - 7y2
Luego. la pendiente de la recta tangente a la superficie en el punto Po es
14 7
m
T
:= f, (-2; 2)
Y
= v16
--:=--
2
Por consiguiente, la ecuación vectorial de la recta tangente a la curva de
intersección de la superficie z = f(x; y) con el plano x := -2 es

LT : (x; y; z) = (-2; 2; 4) + t (O; 1; -~), t E IR/.

Ejemplo 5.- Una recta tangente trazada a la superficie


xy
f(x;y) = e Xsen (6rry) - 2x 3 - arccot(xy) - 1 + X2
en un punto donde Y := 1 está en un plano paralelo al plano YZ y tiene pendiente
-127[. Encuentre la ecuación del plano.
Solución
I 1/ no
x x
I y (X; y) = e xsen (67rY).X cos(6rry) (6rr) + ---
1 + x2y2 1 + X2 '
1111¡¡nees la pendiente de la recta tangente a la curva de intersección de la
.lIpcrficie z = [(x; y) con el plano paralelo al plano YZ (x = xo), en el punto
¡'II( '(o; 1; [(xo; 1)) es
X Xo
tnT = [y(xo; 1) = 6rrxo + - -o2 - --2 = 6rrxo
1 + Xo +
1 Xo
1) Ido que la pendiente de la recta tangente es -12rr, entonces
/liT = 6rrx o = -12rr:::=:} Xo = -2
!iIH tanto, la ecuación del plano paralelo al plano YZ es
!) :x =-2

I Il'lIIplo 6.- Encuentre los puntos de la superficie [(x; y) = xy(l - x - y)


d'Hllle el plano tangente es paralelo al plano coordenado XV.
~1I11Il'ión

el plano tangente es pa.talelo al plano XY, entonces su vector normal es


( 1I1ll0

I 1I,Ik lo
- .,.
al vector k = (O; O; 1). Luego, se tiene

rJI(x;y)
iJ-;- = y(l - 2x - y) =. a[(x;y) = x(l - x- 2y) = O
ay

I I csolver estas ecuaciones simultáneas enc~ramos que los puntos donde se


1II111:r1l las derivadas parciales son (O; O), (1/ 3~~1 /3), (1; O) Y (O; 1). Luego,
Ir 1 cuatro planos tangentes horizontales a la superficie en los puntos

1 1 27
((); O; O), ( 3";3"; 1) ,(1; O; O) Y (O; 1; O)

X2 y2 Z2
Implo 7,- Considere el hiperboloide de dos hojas 4 -4 -4 = 1

1)I Ill.lIentre el plano tangente al hiperboloide en el punto A (-6; 2;...[28)


In Il.dle la ecuación vectorial de la recta normal al hiperboloide en el punto
.I( 6; 2;...[28).
, ) I )1'll'Il1line los puntos sobre el hiperboloide en donde los planos tangentes son
p.I/,lIclos al plano Q: 2x + y + z = O.
ttlllriún
1) J h 1" ecuación del hiperboloide, se obtiene

/, !(x;y) = ,jx 2 - y2 - 4
Como las derivadas parciales de f con respecto a x e y son
-y
fy(x;y) =J 2 2
X - Y -4
entonces
-3 -1
fxC -6; 2) =..J7 y fy( -6; 2) = ..J7
Luego, la ecuación del plano tangente al hiperboloide es
3 1
PT : - ..J7 (x + 6) - ..J7 (y - 2) - (z - ill) = O ~ PT : 3x + y +..[7z + 2 = O
b) La ecuación vectorial de la recta normal al hiperboloide en el punto A es

LN : (x; y; z) = (-6; 2; ill) + t(3; 1; ..[7). t E IR\.

c) El vector normal del plano tangente al hiperboloide en el punto Po (x o; Yo; zo)


es

= JX 2 - 1Yo2 - ( ±Xo; +Yo; - 1X5


. - y~ - 4)
o 4 ~
El vector normal del plano Q es N; = (2; 1; 1)
Como el plano tangente es paralelo al pI 9 Q, entonces sus normales son
~-'l
paralelos. Luego, se tiene
r ¡ k
Ñ x NQ = ±X
o +Yo -JX5 - y~ - 4
2 1 1

= (O; O; O)

Al resolver la igualdad. se obtiene Xo = +2yo • Yo = ±.,fi


Luego. los puntos del hiperboloide donde el plano tangente es paralelo al plano Q
son: B( -2.,fi;.,fi;.,fi) y C(2.,fi; -.,fi; -.,fi)
X2 y2 Z2
Ejemplo 8.- Demuestre que el plano tangente al elipsoide 2" + -b2 + 2' = 1 en
a e
xox YoY zoz
en un punto (xo; Yo; zo) tiene por ecuación Q: -2
a
+ -b
2 + -e2 = 1
1I1111'jún

1'111111'10 consideremos la parte superior del elipsoide, es decir z >O

I 11 ' ',o, se tiene


c2x
J~(x;y) = - - 2 - '
a z
1 l'ul<lción del plano tangente a la elipsoide en el punto Po (xo; Yo; zo) (zo > O)

I'{: {x (xo; yo)(x - xo) + {y(xo; Yo)(y - Yo) - (z - zo) = O


C2xo c 2yo
<=) PT : - - (x - xo) - - 2 (y - Y ) - (z - z ) = O
a2zo b z0 o o
xox YoY zoz
<=)PT : -
a2
+-b 2 +-2e
=1

l' Il'lIIplo 9,- Halle la pendiente de la recta tangente al paraboloide


r. / (x; y) = X2 + 8y2 en el punto A(2; 1; 12), en las direcciones de los ejes X
V Ilspectivamente,
!oo.1I I lIl'jéln

1) I 11 la dirección del eje X, la pendiente de la recta tangente es


/,(x;y) = 2x
I 11 l'i punto A(2; 1; 12), la pendiente de la recta tangente en la dirección del eje
' l" .,

/,(2;1)=4
11) 11 la dirección del eje Y. la pendiente de la recta tangente al paraboloide es
I
J¡.(x; y) = 16y
I lIl:go. la pendiente de la recta tangente en el punto A (2; 1; 12) es
1\.(2; 1) = 16
I N I ERPRETACIÓN DE LAS DERIVADAS PARCIALES COMO RAZÓN
1)1 ('AMBlO

{(x; y) una función de dos variables con dominio D !;; iRl,2 tale~ que
1 Z -

Il\;Y) y {y(x;y)existen\f(x;y)ED,Entoncessetiene:
<J{(x;y) az
11) ax = ax mide la razón de cambio de la variable dependiente z con
n.:specto a la variable independiente x, dejando la variable Y constante
(o fija),

11)
iJf(x;y) az mide la raZÓl, de cambio de z con respec:o a y. dejando
ay ay
1,1 variable x constante (o fija ).
Ejemplo 10.- Suponga que una placa metálica delgada de forma rectangular se
calienta irregularmente, de forma tal que la temperatura en cualquier punto (x; y)
de la placa es
T(x; y) = 4x2y + y
Además, suponga que x e y están medidas en metros y la temperatura T en grados
Isius. ¿Cómo varía la temperatura T en el punto (2; 3) cuando y permanece fijo
en y = 3? ¿Qué significa esto?
Solución
Cuando y permanece fijo, la derivada parcial de T con respecto a x es
Tx(x; y) = 8xy
Luego, la rapidez de cambio de la temperatura T en el punto (2; 3) es
Tx (2; 3) = 48°Cjm
Por consiguiente, cuando y = 3 (constante) y x = 2, la temperatura de la placa
aumenta a razon ae 1f't por caba metro oe llUmertlu 1;;/1 ...t.

Ejemplo 11.- Se construye una caja rectangular cerrada de manera que su


volumen sea 36 pies cúbicos. El costo del material de la tapa y de la base es de
.(-)-1..

SI. 10 el pie cuadrado, el del íltajerial para las partes de enfrente y de atrás es de
SI 9 el pie cuadrado y el material para los otros lados es de S/.7 el pie cuadrado.
a) Determine la función de costo C(x; y), donde x e y son las medidas del largo y
el ancho de la base de la caja respectivamente.
b) Calcule Cx (3; 4) y Cy (3; 4) e inte~e los
resultados. ·'b'r_

Solución I
a) El volumen de la caja rectangular es )-------
,,/
36 /
V = xyz = 36 ~ z= -
r_~ x
De acuerdo a los datos del problema, el costo del
material para construir la caja es
C = 10(2xy) + 9(2xz) + 7(2yz) = 20xy + 18xz + 14yz
Al reemplazar la expresión de z en el costo C, se obtiene
36)
C(x;y) = 20xy + 18x ( -xy + 14y (36)
- = 20xy + -648 504
+ -,x,y >O
xy y x
b) Las derivadas parciales de C con respecto a x e y son
504 648
Cx(x; y) = 20y - -2 Y Cy(x; y) = 20x - - 2
X Y
Luego,
504
CxC3;4) = 80 - 9 = 80 - 56 = 24, Cy (3; 4) = 60 - 40,S = 19,5
1'111 111110, cuando el lado de la base de la caja de medida x es 3 pies y el lado de
1II tlld,1 Y se mantiene constante en 4 pies, el costo de construcción de la caja
1111111 11101 a una razón de SI. 24 por cada pie de aumento en x.
l ' 1I1.1I1l:ra similar, cuando el lado de la base de medida y es 4 pies y el lado de
11 1' dldol x se mantiene constante en 3 pies, el costo de construcción aumenta a una'
, 1/ 1111 dc SI. 19,5 por cada pie de aumento en y.

l' 1'lIIfllo 12.- Se lanza un nuevo producto al mercado. El volumen de ventas V del
1" "dllclO se incrementa como una función del tiempo t medida en meses y de la
1!!lld.IU de e nuevos soles gastada en la campaña publicitaria que está dada por
V V (t; e) = 400(6 - e-o.oo2C)(1 - e-t)
Ikule V, (1; 500) Y !'c(1; 500) e interprete el resultado.
~1I 111 l'i6 n
VI (t ; e ) = 100(6 - e-O.002 C)(e-t ) y !'c (t; e) = O,8e-O.002C(1 - e -t)
I 11 ' ~O, se tiene
1'1 (1; 500) = 100(6 - e - l )(e - l ) = 207,1 94
= 0,8e- l (1 - i';~l) = 0,186
1',.(1; 500)

Illq'O, Vt (1; 500) = 207,194 significa que después de un mes ( t = 1) de haber


Il lllilllo el producto al mercado y mantener constante el gasto en publicidad en SI.
00, d volumen de ventas aumenta a una razón e SI. 207,194 en cada mes.
II lI ilarmente, !'c(1; 500) = 0,186 significa q ~"cuando se ha gastado SI. 500 en
IlIh hcldad en un mes (t = 1 fijo), el volumen d'e ventas aumenta a una razón de
" 0, 186 por cada sol de aumento en publicidad.

e
I 1I'IIIplo 13,- Sea la curva de intersección del paraboloide z = 12 - X 2 - y2
1111 ce p(ano x = z.
1) Ilalle la ecuación vectorial de la recta tangente a la curva e en el punto

11 ( 2; 2;4)
,,) Ilalle la ecuación del plano tangente a la superficie
X2 y2 .
, {(x; y) = '6 +"8 que es perpendicular a la recta tangente obtenida en a).
S"llIdón
\) \1 parametrizar la curva e en términos de y = t, se tiene
2
(1: a(t) = (2; t; 8 - t )
AsI, para t = 2, se obtiene el punto A(2; 2; 4)
luego, se tiene
a '(t) = (O; 1; -2t) Y a ' (2) = (O; 1; -4)
Por consiguiente, la ecuación vectorial de la recta tangente a la curva e en el
punto A es
Lr: (x; y; z) = (2; 2; 4) + seO; 1; -4), s E ~

x y
b) Como fx(x; y) = '3 y fy(x; y) = ¡, entonces el vector normal del plano
tangente a la superficie z = f(x; y) en el punto Po (xo; Yo; f(x o; Yo)) es

Puesto que los vectores Ñ y a= (O; 1; -4) (vector dirección de Lr) son
paralelos, entonces se tiene
-
N=ka~
~ (XO Yo
3;4;-1 ) =k(0;1;-4)~xo=O,yo=1

Por consiguiente, la ecuación general del plano tangente a la superficie en el

punto Po (O; 1;~) con vector normal Ñ = (O; ~ ;-1) es


Pr : O(x - O) + ~ (y -"'~) - (z -~) = O ~ Por: 2y - 8z - 1 = O

Observación 3.- (Continuidad y Derivabilidad Parcial). La existencia de las


derivadas parciales de una función en un punto no garantiza la continuidad de la
función en dicho punto. Por ejemplo, para la ~ión
) (O', O)·",,--
12xy ,S1. ( x,.y"*
f( ) =
x;y x2 +y2
{
O, si (x;y) = (0;0)
las derivadas parciales con respecto a x e y existen en el punto PoCO; O): sin
embargo, f no es continua en (O; O).
La razón por la que una función puede tener derivadas parciales y no ser continua
en un punto es debido a que, la existencia de una derivada parcial depende del
comportamient~ de la función a lo lart,o de un camino lint:al, mientras que la
continuidad depende del comportamiento de la función a lo largo de todas las
trayectorias que pasan por el punto.

2
X (y - 4) .
Ejemplo 14.- Dada la función fex; y) = x +y ,51 X + Y "* O
{
O, si x +Y = O
a) Analice la continuidad de f en el punto A( -4; 4)

af(-4;4) af(-4; 4) . .
b) Halle y ay , SI eXisten
ax
~ ulllción

11) Sean TI={(X;y)E~2 /X=-4} y T2={(X;y)E~2/y=4} dos


trayec torias que pasan por el punto A(-4;4) . Los límites sobre estas
trayec torias son:

Sobre TI : lim f(x; y) = lim f( -4; y) = lim 16(y - 4) = 16


(x ;y)-+ ( -4;4) y-+4 y-+4 Y- 4 .
Sobre T2 : lim f(x; y) = xlim f(x; 4) = lim O = O
(x ;y)-+(-4 ;4) -+ -4 x~-4

Por tanto, por la regla de las dos trayectorias, el límite no existe.


Luego, f es discontinua en el punto A(-4; 4).
11) 1\ 1co nsi derar la definición de la derivada parcial, se tiene:
IxC -4; 4) = lim f( -4 + h; 4) - f( -4: 4) = lim(O) = O
h-+O h h -+ O

f(-4;k+4)-f(-4;4) 16k
l y (-4;4 ) = klim
-+ O
I( .
= kIim -
-+O k
= 16
Por ta nto, las derivadas parciales de f en el punto A( -4; 4) existen.

I Il' JIIplo 15.- Dada la func~n f(x;y) = Ix2 - 4x + y2 - 6y + 41, halle los
IHl lltos en los cuales fy(x; y) no existe.
"'0111 l'ió n
,\ IUlllsiderar la definición de valor absoluto, se tiene
x2-4x+y 2 -6y+4,si(x...· 2 ) 2+(y-3) 2~ 9
I ( x'y ) ={ ~
, _(X2 - 4x + y2 - 6y + 4), si (~- 2)2 + (y - 3) 2 < 9
I derivada parcial de f con respecto a y es
2Y - 6, si (x - 2)2 + (y - 3)2 > 9
/y(x;y) = { 6-2y,si(x-2)2+(y-3)2<9
,\hora, analizamos la existencia de fy(x; y) en los puntos sobre la circunferencia
(12)2 +(y-3)2=9 .
l,1 "o(xo; Yo) un punto sobre la circunferencia (Fig. 3.3). Entonces:

. )-) - l' f(xo; Yo + k) - f(x o; Yo)


I ,xo
«( ,Yo - 1m
k-+O-
k
. -2kYo-e+6k
= k-+O-
hm k
= 6 - 2yo
. )+) - l' f(xo; Yo + k) - f(xo; Yo)
/v c(xo , Yo - 1m
k-+O+
k
Flg 33
2kyo +e - 6k = 2y
= lim - 6
k -+ O+ k o
Por consiguiente, ¡y (xo; Yo) existe si las derivadas parciales laterales son iguales.
esto es
6 - 2yo = 2yo - 6
De donde resulta Yo = 3. Al sustituir este valor en la ecuación de la
circunferencia (x - 2)2 + (y - 3)2 = 9, se obtiene
Xo = -1 ó Xo = 5
Así, ¡y (xo; Yo) existe en los puntos (5 ;3) Y (-1 ;3). En los demás pumas de la
circunferencia (x - 2)2 + (y - 3)2 = 9, ¡y(x; y) no existe.

EJERCICIOS

1.- Halle las primeras derivadas parciales de las siguientes funciones:


a¡ a¡
a) f(x; y) = X2 sen2y R. ax = 2x sen2y. ay = X2 sen 2y

y2 a¡ a¡
';3- = x y 2y In x
2 2

e~~)
b) ¡(x; y) = x R. ax = y2Xy -1

,) f (x; y) ~ x'e- Y + In + se:' (rrxy)

d)f(x; y) = In (~) + arctan (:2) + arcsen (xy)

e) z = In ( ) X2 + y2 - x) az
R - = % -
az <z 2x
) X2 + y2 + X . ax ) X2 + y2 • ay y) X2 + y2
f) U = eX / Y + e Z / Y + sen (2y - z + x)
X2 - y2)
g) ¡(x; y) = arctan
(
X2 + y2 + arcsen ( 1 +
X )
Y

h) ¡(x; y) = xye X2
+Y
2
+ In (X--- 1)
y-1
Y
i) ¡(x; y; z) = X2 +xyz
y2 + Z2
+ e XYz + arctan (3X
- )
Z2
z2 1
j) ¡(x; y; z) = xlxZ . 1+cos2t dt + yz3
k) ¡(x; y; z) = (x 2 + y2 + Z2) In(~r-x-"'2-+-y"""2:;--+-z-"'2) + xz 2 - yx 2 + zy2

2.- En los siguientes ejercicios, determine las derivadas parciales indicadas en


caso de que existan.
1 + cos(rrxy) .
a) [(x; y) = x+y ,SIX+y*O, [x(l;-l) Y [y(l;O)
{
° ,
x2y2.
si x+ y =
x
°
b) [(x; y) = y + eX ,SI Y * .-e , [xCO; -1) y [y(O; 1)
{
°,
X3 _ y3
si y = -ex
.
c) [(x; y) =
{
X2 + y2 ,SI (x; y) *
(O; O), [xCO; O) Y [y(O; O)
0, si (x; y) = (O; O)
X3 + y2
2+ *O, [xC-1;1)Yf~(-1;1)
si y x
d)[(x;y)= y2+X ,
{
°, si y2 +x = °
au au
," Si u = In(x 2 + xy + y2). Pruebe que x ax + y ay = 2

"Ill
y
= -
x
z
+- +- ,
x
x
y
pruebe'~e
au
x- + y- + z-
iJx
au
ay
au
az
= °
, au au
SI 11 = y2 + tan(ye 1fx ) , pruebe que X2 - + y- = 2y 2
ax ay .

(, , i 11
z
= xy + yz + xz , pruebe que x;:¡-
ux
au "1h1
+ y;:¡- + z;:¡- + u =
uy uz
au
°
e XYZ au au au
Si 11 = eX + eY + e , pruebe que -ax + -ay + -az = u(xy
Z
.
+ xz + yz - 1)

( duill! la pendiente de la tangente a la curva de intersección de la superficie


¡', J36 - 4X2 - 4y2 Y el plano x = 2 en el punto Q(2; -1; 4) R. I

I llId Il:l:ta tangente trazada a la superficie z = 9 - y2 - yx Z en un punto en el


I l' , octantl! donde x = -2 está en el plano paralelo al plano YZ y tiene

1111l!111l: 8. Encuentre la ecuación del plano. R. y = -2

I Il t JII I ,11 :111" l:nlllina hacia arriba a lo largo de la curva dada como la
IIlt\1 ' ll'inll dl: la superficie z = x ~ + xy3 + 12 con el plano x = 1. En el
pl/Hltl (1 ; ::! ; ;,), salio por la recta tangente. ¿En dónde tocó la araña al plano
" R. (1; O; 29)
11.- Una recta tangente trazada a la superficie
2 xy 2
z = e2ycosl7rrx) + 4 y s - arctan(2 xy 2) + - - -
1 + y4
en un punto donde x = -1/2 está en el plano paralelo al plano XZ y tiene
pendiente 14IT. Encuentre la ecuación del plano. R. y = -1

12.- La intersección del plano y = 1 con la superficie z = X3 y + Sy 2 es una


curva C. Si se traza la recta tangente a e en el punto donde x = 1, halle el
punto donde dicha tangente corta al plano x = O. R. (O; 1; 3)

13.- Considere una esfera con centro en el origen y radio 13 . Una recta tangente
trazada a esta esfera en un punto en el primer octante donde x = 3 está en el
plano paralelo al plano XZ y tiene pendiente -1/4. Encuentre la ecuación del
plano. R. y = 4

14.- En cada uno de los siguientes ejercicios. halle la ecuación del plano tangente
y de la recta normal a cada ..1lna de las superficies en e1 punto indicado.

a)z=3-
X2
-
y2 (.>..
, Po 2;2;36
83) b)z=xlny,(l;l;O)
9 16
c) z = )4 - X2 - y2, Po(l; 1; v'2)
d)z=3x2+y2+2.
e) z =e 2X
cos 3y ,
t)z=ln()x2+y2), Po(-3;4;lnS)
Po (-1;2;9)
Po(l; IT /3; -e 2 ) -<
.~
R.6x-4y+z+S=0
R. 2e 2 (x - 1) + z + e2 = O

15.- Halle los puntos de la superficie donde el plano tangente es paralelo al plano
coordenado XY .
X2 y2 Z2
a) 4 + 9 + 1 = 1

c) z = x 3 - 12xy + 8y 3

16.- Halle la ecuación del plano tangente a la superficie z = 4xy - x 4 - y4 que


es paralelo al plano Q: 8x - 8y + z + 28 = O R. 8x - 8y + z - 10 = O

17.- Encuentre el ángulo entre la recta L = {( -2; S; 12) + t(4; 1; -3) / t E ~} Y


la normal a la esfera X2 + y2 + Z2 = 121 en el punto de intersección de la

'''lo y l. "foc. R. '0' e ~ ±~


1X ¿En qué puntos del gráfico de la ecuación X2 + 4y2 + 16z 2 - 2xy = 12,
son los planos tangentes paralelos al plano XZ? R. (2; 2; O) Y
( 2; -2; O)

11) ,- Ilalle un vector tangente a la curva de intersección de las superficies


\"2 _ 3xz + y2z = 1 Y 3xy + 2yz + 6 = O en el punto (1; -2; O).

() Demuestre que el plano tangente a la esfera X2 + y2 + Z2 = 1 en un punto


(.\0; Yo; zo) de la esfera (zo > O) tiene por ecuación xxo + yYo + zZo = 1

Lncuentre las intersecciones con los ejes coordenados de cada plano tangente
iI la superficie X2/ 3 + y2/3 + Z2/3 = a 2/ 3

, ' ,- Prucbe que el tetraedro acotado por los planos coordenados y cada plano
9
tangente a la superficie xyz = a 3 es de volumen constante, R . V = -2 a 3

Ilalle sobre el cilindro (;~+ JlY+ (y - Z)2 = 4 el lugar geométrico de los


plintos en los cuales la normal es paralela al plano XY. R. Y = x, x + y = ±2

1 I Detcrmine el valor de m para que el plano x - 2y - 2z + m =O sea


1 IIl!.(ente a la superficie de ecuación X2 + ~+ 16z - 144 = O
2

''!¡l>

- I a temperatura T de una placa rectangular está dada por


t(\;y)=4 xy 2(S-X)(S-y), si O~x~S,O~y~S, En (4;2),
dl'lcrmine la razón de cambio de T. a) con respecto a x b) con respecto ay.

' r, 1..1 ("unc ión de utilidad U = f(x; y) mid,e la satisfacción (utilidad) que
l'lll'Uentra una persona al consumir dos productos x e y , Supongamos que
/1 5x 2 - xy + 3y2
,1) ( aleule la utilidad marginal con respecto al producto x (Ux(x; y))
11) Determine la utilidad marginal con respecto al producto y (Uy(x; y))
l ) ( 'liando x = 2 e y = 3, una persona ¿debe consumir una unidad más de x o
,k y para tener más utilidad?

1.1I)1"ieantc de pistones para autos estima que su producción total en miles


1)11
ti IlIlldades está dada por P(x; y) - lSX 2/ 5 y3/5, dondc "x" es el número de
IIl1lll.Hk~ de fucrza de trabajo e "y" es el númcro dc un idades de capital
111 di ,,,do
a) Encuentre el número de unidades producidas cuando se utilizan 32 unidades
de fuerza de trabajo y 7776 unidades de capital.
b) Encuentre e interprete Px (32; 7776) Y Py (32; 7776).
c) ¿Cuál sería el efecto aproximado sobre la producción de incrementar a 33
unidades de fuerza de trabajo mientras se mantiene el capital en su nivel
presente?
d) Suponga que las ventas han sido buenas y la administración quiere
incrementar el capital o bien la fuerza de trabajo en una unidad . ¿Qué
opción dará un mayor incremento en la producción?

28.- Después que un nuevo producto se ha lanzado al mercado, su volumen de


et + 4S0
ventas V (en miles de unidades) está dado por V = .~ ,donde t es el
ve +t2
tfempo (en meses) desde que el producto fue introducido por primera vez y e
la cantidad (en cientos de nuevos soles) gastada cada mes en publ icidad.
a) Calcule Vc(e; t)
b) Use el resultado de la~part.e a) para predecir el número de meses que
transcurrirán, antes de que e volumen de ventas empiece a descender, si la
cantidad destinada a publicidad se mantiene fija en SI. 9000 por mes.
R. 18 meses

29.- Una compañía que fabrica computadoras ha<~tenninado que su función de


4
producción está dada por P(x; y) = SOOx + 800y + 3x 2 y - x3 - Y4 ' donde
x es el tamaño de la fuerza de trabajo (en horas de trabajo por semana) e yes
la cantidad de capital (en unidades de SI. 1000) invertido .
Encuentre J~(x; y,) y' Py(x; .y) cí.tBada x == 50 y' .V == 2D oC interprete las
resultados.

30.- La función de costo de la empresa SAJ ITA S.A. que produce dos tipos de
productos A y B es C(x;y) = SOlnx + 40 lny + lSy2 + 12x 2 , donde x e y
son las cantidades producidas de tipo A y B respectivamente.
a) Encuentre el costo aproximado de producir 50 de tipo A y 20 de tipo B.
b) Halle CASO; 20) Y Cy(SO; 20) e interprete los resultados.
c) Suponga que las ventas de los productos han sido buenas y la empresa
quiere incrementar la producción del producto de tipo A o del tipo B en
una unidad ¿Qué opción dará el menor costo de producción?
1, DERIVADAS PARCIALES DE ORDEN SUPERIOR

I o mismo que sucede con las derivadas ordinarias de una función de una variable
I <; 11 es posible encontrar derivadas parciales de segundo, tercero, cuarto y en
'llH.: ral de orden n de una función de varias variables.
V,lInos a empezar por denotar las derivadas parciales de orden superior de una
IlIlIl ión de dos variables. Luego, se generaliza esta idea para funciones de n
vlIla bles .
• ,('.1 7. = [(x; y) una función de dos variables con dominio el conjunto D e !R{2

I 'lIesto que las derivadas parciales de primer orden de [

az
ax = [x(x; y) = Dd(x; y)
az
ay = [y (x; y) = D2f(x; y)
,0 11 también funciones de dos variables, entonces las derivadas parciales de estas
1IIII1Iones se llaman derivadas parciales de segundo orden de [.
I ,t.1S segundas derivadas de [ stm I!uatro y se denotan por

a2 z
iJx 2 = [xxCx;y) = Du[(x;y)
a2 z
ay2 = [yy(x; y) = D22 [(x; y)
a2 z
aya x = [Xy(x; y) = Dl2f(X; y)

I 1'0 derivadas parciales [xy(x; y) y [yx(x; y) se conocen como derivadas


p III ¡¡tles m ixras O cruzadas de f.
( IlIIIO las derivadas parciales de segundo orden de z = [ex; y) son funciones de x
, l'lltonces se puede derivar nuevamente para obtener las derivadas parciales de
I 1\ 11 oruen de f y así sucesivamente hasta el orden n.

I 11' 111 "lo 15.- Ilalle las derivadas parciales de segundo orden de
I ( \ , 1') 2 xy 2 - 3x + 3x2y2 y calcule el valor de [xy(l; -2)
SlIl lll'iúlI
I l i S dl.!llvadas parciales de primer orden de [ son
fx(x; y) = 2y Z - 3 + 6xy z y fy(x; y) = 4xy + 6x Zy
y las derivadas parciales de segundo orden de f son

fxx(x;y) = 6yZ , fyy(x; y) = 4x + 6x z


fxy(x;y) = 4y + 12xy , fyxCx; y) = 4y + 12xy
Luego,
fxy(1: -2) = -8 - 24 = -32

Teorema 1.- Si z = f(x; y) es una función continua en un punto P(x; y) y las


funciones derivadas parciales fxCx; y),fy(x; y),fXy(x; y) y fyx(x; y) están
definidas y son continuas en la vecindad del punto P, entonces se cumple:

fxy(x; y) = fy(x; y)

Observación 4.- Si la función r


D e IR?z --7 IR? Y sus funciones derivadas
parciales Dd(x; y), Dzf(x; Y}:~f(x; y), D2d(x; y), Dl21 f(x; y), D21 d(x; y)
son continuas en el punto Po (x o; Yo , entonces se cumple

En seguida vemos cómo estos conceptos de d~das parciales de orden superior


para funciones de dos variables se generalizan a1\lnciones de n variables.
Sea z = f(x l ; X2; ... ; x n ) una función de n variables con dominio el conjunto
De IR?n.
Como la derivada parcial de f con respecto a la i-ésima componente
(i = 1,2, ... , n)
af(x l ; ... ;xn )
Di/(x 1 ; XZ; ... ; Xn ) = fx '(x I ; ... ; Xn ) =
I
a
Xi
es una función de n variables, entonces las derivadas parciales de estas funciones
se llaman derivadas parciales de segundo orden de f y se denotan por

aZ f(x l ; .. . ; x n )
Dijf(XI; ... ; x n ) = fx.x .(xI ; ... ; x n ) =
I )
a a
Xj Xi
Vi = 1;2, ... ;n, Vj = 1;2, ... ;n

Las derivadas parciales de la función Dijf: D e IR?n --7 IR? con respecto a la k-ésima
componente se denominan derivadas parciales .de tercer orden y se denotan por:
z f(x ; ... ; x ) a l n
Dijkf(XI;",;Xn) =fX·X'Xk(XI; .. ·;x,J
I )
= a a a
Xk Xj Xi
Vi = 1,2, ... ; n;
j = 1,2, ... , n ; k = 1,2 . . , n
I I1 lorllla simi lar, se puede c<?ntinuar en hallar las derivadas parciales de orden n
" l. el1 caso existan.

I Il"Il1plo 16.- Dada la función [(x; y; z) = e XYz - e-Y cos(xz), halle


f .. (x; y; z) y [ZXY(x; y; z).
ollldón
I ,1\ derivadas parciales de primer orden de [ son

I (r; y; z) = yze xyz + ze-Y sen (xz) y fz(x; y; z) = xye xyz + xe- Y sen (xz)
I IIl:g0, las derivadas parciales de segundo orden de [ son:

I~y(x;y; z) = ze xyz + xyz 2e XYZ - ze-Ysen (xz)


II.X(x; y; z) = ye xyz + xy2 ze xyz + e-Y sen (xz) + xze- Y cos(xz)
I 1I1.rlmcnte, las derivadas parciales de tercer orden de [ son:

l.y~(X;y;z) = e XYz + 3xyze~~Z +x2y2z2eXYZ - e-Ysen(xz) - xze- Y cos(xz)

1" y(x; y; z) = e XYz + 3xyze xyz + x2y2z2eXYZ - e-Y sen(xz) - xze- Y cos(xz)

EJERCICIO

l. Ifalle todas las derivadas parciales de ~undo orden de las siguientes


funciones:
x +y )
b) z = arctan (
1-xy
x
e) z = u = xy + yz + zx d)[(x;y) = . - -
x+y
e) [(x; y) = e X/Y f) z = X2 - 4y + In(Jx 2 + y2) - 3 arctan (~) ,x> 0_

g) [(x; y) = x In (:2) h) z = e yx2 + In(cos(x - y))

1 Vcrifique en cada caso que D12 [(X; y) = D2d(x; y).

a) [(x; y) = x 4 + 4x 3 y - 3x2y2 + 6 xy 3 + 9y4


y2
b)[(x;y) = e XY senxcosy c) [(x; y) = xe- + xsecy

d) [ex; y; z) + zX)
= In ( 11 + - e
XY
x-y
e) [(x; y) =-- f) [(x; y) = xe XY
x+y

. [YX(O; O)
3.- SI [(x; y) = sen (Srrx + y) + cos(x - Srry), calcule [XX (O; O)

4.- Una función f : ~2 -) ~ se llama armónica si satisface la ecuación de Laplace

a2 [2 + -;-;¡-
-a a2 f = o. 1> ruebe SI. las siguientes
.. f '
uncIOnes . .
son annonlcas
x (¡y
a) [(x; y) = x:ly· xy3 b) [(x; y) = e-x sen y

c) u = c:\ se n y + In(x 2 + y2) + x 3 - 3xy2


1
e) [(x; y) = --,==o;:==:;:
~X2 + y2

5.- Si H = 11 rOS/llleX ·1 aL)] + B sen [n(x - at)] . Pruebe que


a2u ¡j <1I •
t ( X"
a,
-a2 = (12 -) " donde 11, 111,/1 ya son constantes.
'''>

6.- Para la función lL

Pruebe que las siguientes funciones satisfa n la ecuación de Laplace.


a) u = (x 2 + y2 + Z2)-1/2 b) z = eX se1fJ:".y + eY sen x

7. - SI' [(x,' y) = (y + ax)2 e y+aX . Pruebe que Jxx


r -- a 2Jyy
r

2 2
a u ( a z az
9.- Si u = eX + eY + e Z
, pruebe que - - - e Z -- + - - az)
-
axay - axay ax ay

a2 u au
2
10.- Si u = e x- at cos(x - at)

pruebe que - -
at 2
= a 2- -
ax2
X a 2u a 3u a 3u
11.- Si u = ye + xe Y • pruebe que axay = ax2ay + ay 2ax
XY (X 2 - y2) . 2 2
12.- Si[(x;y) = X2+y2 ,SlX +y *0
{
O. si (x; y) = (O; O)
pruebe que f12 (O; O) = -1 , f21 (O; O) =1

13.- Dada la función F(x;y) = Ax 3 + 3Bx2y + 3Cxy2 + Dx 3 . Determine que


relación debe existir entre los 'coefi cientes A, B, C y D para que Fxy - FxxFyy
sea un cuadrado perfecto.

1
14.- Dada la función z = Sx 5 - 2X3 + 2Sx + ax3y2 + bxy4 + cxy2
82z 82 z
a) Determine los valores de a, b y e de modo que 8X2 y 8y2 sean iguales

y de signos opuestos.
b) Halle los puntos de la superficie representativa de dicha función en los que
el plano tangente es horizontal.

15.- Sea la función f(x;y) = e tlx + by g(x;y). Si gAx;y) = gy(x;y) = 1. l falle


los valores de las constantes. a y b, tales que fAx; y) = fy(x; y) y
~%.:
1 + fxy(x;y) = a + fyx(x;y) R. a = b = 1

=r 9lX ;y;Z)
1Ü.- Sean g(x; y; z)
zJXY
= -2- Y f(x; y; z) =:,¡,. sen (t2)dt

8 2f(2; rr; 1) 8 2 f(2; rr; 1) 8 2 (C2;'ft; 1)


lIalle y

v =-2
r r . l" 3/'
R. -32'
- --/?rr)-
2 \.~
L
.
O

1 x
I ¡' Para k una constante positiva y g(x; t) = -2 ~. sea
vkt
9 (X ;t) k82f 8f
((x; t) = r
Jo
e- u2 duo Pruebe que
8X2 8t

y
(eX+eY+ ,x , ,si (x; y) '1"(0;0)
lti Dada la función f(x; y) = ) X· + y.
l 2, si (x;y) = (0;0)
a~f(O;O) 82 f(0;O)
II,ilk v si es que existen.
8x8y
3.3 DERIVADA DIRECCIONAL Y GRADIENTE DE UNA FUNCIÓN DE
VARIAS VARIABLES

En la sección 3.1 hemos determinado la pendiente de la superficie z = f(x; y) en


dos direcciones diferentes: en la dirección del eje X (la pendiente estaba dada por
la derivada parcial fAx; y)) y en la dirección del eje Y (la pendiente estaba dada
por la derivada parcial fy(x; y)).
En esta sección veremos cómo se puede usar estas dos derivadas parciales para
encontrar la pendiente de la superficie z = f (x; y) en una dirección arbitraria.

Definición 5.- Sea f: D e I!lin 4 I!li una función de 11 variables con dominio
De I!lin , tales que Dtf(XI; ... ;xn ), ... , Dnf(XI; ... ;x n ) existen \i(x l ; ... ;xn ) E D.
El gradiente de la función f en el punto (Xl; ... ; Xn ) E D es el vecror

donde íl es el operador nabla.


Geométricamente, el gradiente íl f (Xl; ... ; Xn ) es u~ vecror normal a una curva o
superficie en el espacio en la c~1 se estudia.

Observación 5.-
i) Si z = f(x; y) es una función de dos variables, tales que
íl f(xo; Yo) = (¡x (x o; yo);fy(xo; Yo)) "* O~tonces íl f(xo; Yo)
es un vector
normal (ortogonal) a la curva de nivel de r~eeN:f(x;y) = e) que pasa por el
punto Po(xo;Yo) (Fig. 3.4).

y
¡:¡ = 'ilf(x,:y,)

x
z
Fig . 3.4 Fig.3.5

ii) Si w = f(x; y; z) es una función de tres variables tal que


íl f(xo; Yo; zo) = (¡x (xo; Yo; zo); fy(xo; Yo; zo); fz(x o; Yo; zo)) =1= O,
elllonces 'íJ f(x o; Yo; zo) es un vector normal (ortogonal) a la superficie de
nivel (SN: f(x; y; z) = e) que,pasa por el punto Po (x o; Yo; zo) (Fig, 3.5).

1 Iflllplo 17.- Halle el vector gradiente de las siguientes funciones


01 (.t; y) = 8xy - 2x 4 - 2 y 4 b) g(x; y; z) = cos(xy) + X3 y 3 z 3
ollldón

1) VI Cx; y) = (tx-(x; y);fy(X; y)) = (8y - 8x 3; 8x - 8y 3)


h 'íJ!I(x; y; z) = (fx(x; y; z);fy(x; y; z);fz(x; y; z))
= (-y sen (xy) + 3X 2y 3 z 3; -x sen (xy) + 3x 3 y2 z 3; 3X 3 y 3 Z 2);

y2
1 Iflllplo 18.- Si f(x; y) = xe - 151n(x 2 + y2 + 16), hall e 'íJ [(2; O)
_ I •• _!J.. _

v¡(x; y) = (fAx; y);fy(x; y))


2 30~_ 2 30 Y )
= (e
Y
- X2 + y2 ¡ 16:,2xy e Y
- X2 + y2 + 16
111 o 'íJf(2;0) = (-2;0)

1I un'ma 2.- Sean f, g: D e ~n --? ~ funciones de n variables. entonces 'íJ es un


"1 I ,,11m que satisface las siguientes propiedades:

viI (71) + g(p)] = 'íJf(p) + 'íJ g(p), 'V'p (x 1; oo.: x n ) ED

vrníJ)'- g(p)f= 'íJ[(p) - 'íJg(p)

ti lA lCp)] = il. 'íJf(p)


VI! C/J)g(p) ] = f(p)'Vg(p) + g(p)'íJf(p)

vl!(¡J)l = g(p)'Vf(p) - f(p)'íJg(p) ,si (p) '* O


l/(p) [g(p))2 9

VIU (II))'J = r[f(pW-1Vf(p)

I 1 'llm 'íJ [(p) indica la dirección de máxima razón de cambio de [ en el


1'111110 l.
DERIVADA DIRECCIONAL DE UNA FUNCiÓN DE V ARIAS
VARIABLES
Definición 6.-
Sea f: D e IR\.2 ~ IR\. una función de dos variables con dominio D e IR\. 2, Y sea
u = (u 1 ; u 2 ) un vector unitario en IR\.2.
La derivada direccional de f en el punto ex; y) E D en la dirección del vector
unitario u, es la función de dos variables denotada por
D fe ) r f((x; y) + hu) - f(x; y)
U x; Y = J¡1!;6 h

si este límite existe.

INTERPRETACiÓN GEOMÉTRICA DE LA
DERIVADA DIRECCIONAL

Sea f: D e IR\.2 -7 IR\. una función de dos variables


,tal que Duf(xo; Yo) existe para (x o; Yo) E D Y ü
vector unitario en IR\.2. ""~~
La derivada direccional de f e~i>el puma exo; Yo) E I
~ I •. "
o

D en la dirección del vector unitario U, esto es.


Ftg 36

-f( . ) - l' f( exo; Yo)


Du xo,Yo - 1m
+ hu)) - f(xo; Yo)
h
J¡-->O ~
representa la pendiente de la recta tangente (LT~'4a la curva de intersección de la
superficie z = f(x; y) con el plano perpendicular al plano XV que contiene a la
recta L: (x; y; t) = (x o; Yo; O) + t(xo; Yo; O), t E IR\.

Observación 6.- (Interpretación de la derivada direccional como razón de


cambio)
i) La derivada direccional de f en el punto (x o; Yo) E D en la dirección del vector
u
unitario = (U 1;U2), esto es,
D f( . ) - r f((xo;Yo) + hu)) - f(xo;yo)
u xo, Yo - J¡1!;6 h

mide la razón (o velocidad) de cambio instantáneo del valor de la variable


dependiente z = f(x; y) con respecto a la distancia en el plano XV. medida en
la dirección del vector unitario U.
ii) Si la dirección del vector unitario u
está dado en términos del ángulo que
u
forma este vector con la parte positiva del eje X, esto es, = (cos () ; sen ()),
entonces la derivada direccional de f en cualquier punto (x; y) E D es dada
por
'( , ) _ ' f(x + h cos 8 ; Y + h sen 8) - f(x; y)
/),,/ x, y - 11-+0
hm h '

I sIc límite existe,

1\ 1111'10 19.- Sea f(x; y) = e XY - y2, Halle la derivada direccional de f en


111 IlIi!.:!' punto (x; y) E DI ' en la dirección del vector unitario
1 1
ti C.¡z; - viz)
111 11 l'il\n
, ) ' fe (x; y) + hü)- f(x; y )
= l 1m
/) 11 f( x,y
11-+0 h

f(x +~;y -~) - f(x ;y)


= Iim ' viz -ti
11-+0 h

[
e(x+Jz)(~:Jz) _(y _ ~)2] _[e xy _ y2 ]
= Iim . '" viz
11-+0 h

L'H
( ~Y-~
e..f2 ff - 112) ( Y
2 ----h
X) 2
h--=y
'"" e xy l 1m
= ' ..fi...fi. ' _----"-'./~2_
- l 1m
11-+ 0 1 11-+0 1

1) IIl1kión 7.- Sea f: D e ~2 '-+ ~ una función de n variables con dominio el


'")111110 D e ~n , y sea ü = (ul ;
un) un vector unitario en ~n,
,.. ;

1 I dl'llvada direccional de f en cualquier punto (Xl; X2; .. ,; ~n) E D en la


l!'In del vector unitario ü es la función de n variables dada por

1 'hllill' existe,
Teorema 3.- Sean f: D e ~2 -+ ~ una función de n variables con dominio D,
11 = (UI; ... ; un) un vector unitario en ~n y Vf(P) el vector gradiente de f el1 el
punto P(x1 ; ..• ; x n ) E D, entonces la derivada direccional de f en la dirección del
vector unitario 11 es
af(p) af(p) af(p)
Daf(P) = Vf(P) • 11 = - - U I + --U2 + ... + --Un
aX 1 aX2 aXn

Ejemplo 20.- Calcule la derivada direccional de la función


f(x; y; z) = In(x 2 + y2 + Z2) en el punto Po(2; 2; -4) en la dirección que va de
PI (2; 2; -4) a QI (3; 1; -5)
Solución
Un vector en la dirección indicada es
a= P1 Ql = (1;-1;-1)
y un vector unitario en la dirección de este vector es
_ a (1 1 1)
u(i = "a" = ,.[3; - ...¡¡; - ,.[3
2x"'''' 2y 2Z)
ComoVf(x;y;z) = ( x 2 +y 2 +z 2; x 2 +y 2 +z 2; x 2 +y 2 +z 2 • entonces el

vector gradiente en el punto Po es


1 1 1)
Vf(2;2;-4) = ( 6;6;-3
Por tanto, la derivada direccional de f en el punil;tPo es
1
Dü f(2; 2; -4) = Vf(2; 2; -4) • 11 = í-l
3v3

Ejemplo 2J.- ¿Cuál es el valor de} angulo f) para el cual la deri~'ada direcciona!
de f(x; y) = .J25 - X2 - y2 en el punto (1; 2) es mínimo y cuál es este valor
mínimo?
Solución
=
Al usar el vector unitario 11 (cos 8; sen 8), se
tiene
9(8) = Daf(l; 2)
1 1
= ---=cos 8 - -sen 8
NS v'5
De donde resulta
1 1
9'(8) = r;:sen 8 - r;:cos 8 y
2vS vS ~, 37
1 1
g" (e) = r;= cos e + r;= sen e
2v5 v5
\1 hacer g'(e) = 0, el punto crítico de 9 es e = arctan(2)
I Ilrgo,
,
11' (ill'ctan(2)) =
1 1
r;=cos(arctan(2)) + r;=sen (arctan(2))
2v5 v5
1
=- >
2
°
\ ~í. () = arctan(2) corresponde a un valor mínimo de g.
I'ell lal1to, el valor mínimo de la derivada direccional de f es

I'I{()PIEDADES DE LA DERIVADA DIRECCIONAL

f,g: D e IRl. n -) IRl. funciones reales de n variables, tal que Vf(?) y óg(?)
', \.' .111

e ISlen. TI?(xl; ... ;Xn)ED, y",sea .


u=(uI; ... ;u n ) un vector unitario en IRl. n .
~.

I 1I111llces se tiene:

1) 0li([ ± g) = Dd ± Dug
¡,) O,;([. g) = fDug + gDuf -i'
1) /JI; ( 9f) = gDuf-fDug.
g2
.'
g(?) =r= 0, TI? E fj
,SI
.,<)

el) La dirección del ascenso más rápido de la variable dependiente


/. = f(x l ; ... ; x n ) (o la dirección de máxima razón de cambio de z = fe?))
\.'n el punto ?(x l ; ... ; X n ) E D, se presenta cuando el vector unitario
li = (U I ; ... ; un) tiene el mismo sentido que el vector gradiente Vf(x l ; ... ; x n ).
En esta dirección. el valor máximo de la derivada direccional de f es

1 ) I ,a dirección del descenso más rápido de la variable dependiente


/. = f(x l ; ... ; x n ) lo la dirección de decrecimiento más rápido de z = fe?))
\.'n el punto ?(x l ; ... ; X n ) E D, se presenta cuando el vector unitario
,i - (u l ; ... ; un) tiene el mismo sentido que el vector -v f(x l ; ... ; x n ) .
I 11 esta dirección, el menor valor de la derivada direccional de f cs
De las propiedades d) y e), la derivada direccional de { en la dirección de
cualquier vector unitario ¡¡ del espacio ~n satisface la desigualdad

f) Cualquier dirección ¡¡ = (u l ; ... ; un) perpendicular al vector gradiente


'í/{(Xl; oo.; X n ) es una dirección de cambio cero en {. esto es

D¡¡f(x 1 ; oo . ;xn ) = 'í/{(x l ; oo.;x n ) - ¡¡ = O

g) D_ i¡{(X1 ; oo . ; x,J = -Dü!(x1 ; oo.; x,J

h) Las relaciones de las derivadas parciales de la función z = {(x; y ) con la


derivada direccional de { son:
Dt!(x; y) = 'í/{(x; y) - L = {Ax; y) (L = (1; O))
D~((X:Y) = 'í/((~; 1'1- r = f..,(x; y) Cl = (O; -1))

Ejemplo 22.- La distribución de la temperatura sobre una placa metálica viene


dada por la función
T(x; y) = 10(xe- Y z +ye-5 X
-
2 )2 )

Si una mosca se sitúa en el punto Po(2; O). se pide :


a) Determinar la razón de cambio de la temperatura al desplazarse hacia el punto
Qo(2;2).
b) ¿En que dirección desde el punto Po d~ moverse la mosca para que la
temperatura disminuya lo más rápidamenté"'flosible". Si sigue esta dirección.
¿cuál es la rapidez de cambio de la temperatura?
c) ¿En que dirección desde el punto Po debe moverse la mosca para que la
temperatura aumente lo más rápidamente posible? Si sigue esta dirección,
S-'L:\J. ~,,-I.l'l_':~!!id~z.. dt:. 1J1JJJhin. dt:. la tf!.IlJJferatura?
d) Si la mosca no quisiera apreciar ningún cambio de temperatura, ¿que dirección
debe tomar?
Solución
a) Un vector en la dirección de Po hacia Qo es
b= PoQ; = (O; 2)
Y un vector unitario en la dirección de este vector es
_ b
u¡; = IIbll = (O; 1)
Como 'í/T(x; y) = (10(e- y2 - 2y(x - 2)e- tX - 2 )2); lOe -2xye- Yz + e- lX - 2 )2)).
entonces el vector gradiente de T en Po es
'í/T(2; O) = (10; 10)
Luego, la razón de cambio de la temperatura al moverse en la dirección del
vector b es
Dü ](2;
b
O) = VT(2; O). • 11-b = 10
11) Para que la temperatura disminuya lo más rápido posible, la mosca debe
moverse en la dirección del vector
-VT(2; O) = (-10; -10)
Ln esta dirección, la rapidez de cambio de la temperatura es
ID üVT T(2; 0)1 = 1-IIVT(2; 0)111 = 1-10~1 = 10~
) ".Ira que la temperatura aumente lo más rápido posible, la mosca debe moverse
en la dirección del vector
VT(2; O) = (10; 10)
I n esta dirección, la rapidez de cambio de la temperatura es

[D üv J(2; 0)[ = IIIVT(2; 0)111 = [10~[ = 10~


1) "\1m que no haya cambio en la temperatura. se busca el vector unitario
li (u 1 ; U2). tal que

fVT(2; O) • 11 = O~ 10u1+ 10u2 = O (1)


l 111111 = 1 ~ ui + u~ = 1 ... (2)
1\ l resolver las ecuaciones (1) y (2), se obtiene

l'llI tamo, la mosca debe tomar una de las direcciones 111 o 11 2 para no tener
Illll g llll cambio en la temperatura de la placa.

1 IllIIplo 23.- La altura de una montaña sobn; el nivel del mar es dada por la
II.Il IOIl Z = 900 - 2x - 2 y 2. donde x e y medidas en metros son las
2

l"l l\kllada~ este-oeste y sur-norte respectivamente. Un hombre se encuentra en el


!,1I1ll11 ,l(C>; S; zo).
11 ,1\ qllc altura se encuentra el hombre?
1 ~ ., 1 11 qlle dirección desde el punto A debe caminar el hombre para escalar la
1110111.11'" lo más rápido posible? Si sigue esta dirección. ¿cuál es la rapidez de
1/lIhio del hombre? (considere la unidad de tiempo ell segundo).
11111 c<, la dirccción que apunta a la cima de la montaña desde el punto A? Si
I ti ' l· ~ t" dirección, ¿cuál es el valor de la pendiente de la montaña')

I 1 "lImbre se mueve en la dirección sur-oeste. ¿está ascendiendo o


ti ' I,," elldo'!. ¡,cuál es su rapide:. :?
e) Describa el lugar geométrico de los puntos que el hombre debe recorrer, si su
deseo es estar a la misma altura sobre el nivel del mar que en el punto A.
Solución.
a) El hombre se encuentra a la altura de

Zo = [(6; 5) = 900 - 2(36) - 2(25) = 778 metros


b) Como 'í/[(x;y) = (¡xC x ; y);fy(x; y») = (-4x; -4y), entonces el vector
gradiente de [ en A' (6; 5) (Proyección de A sobre el plano XV) es

'í/[(6; 5) = (-24; -20)

Luego, la dirección que debe caminar el hombre para escalar la montaña lo


más rápido posible es

'í/f(6; 5) = (-24; -20)


En esta dirección, la rapidez de cambio de [ es

Düu[(6; 5) = 1I'í/f(6; 5)11 = -.)976 == 31,24


't:~
Por consiguiente, el hombrte.st:t subiendo con una rapidez de 31,24 m/seg

c) Como la superficie de la montaña tiene la forma de un paraboloide elíptico con


vértice en el punto V(0;0;900), entonces la dirección que apunta a la cima de
la montaña es dada por el vector que va ~unto A'(6; 5) hacia el origen de
coordenadas, esto e s < " ,.".
a= A'O = (-6; -5)
Y el vector unitario en esta dirección es
. a 6 5
Uii = \\un = (- ..jf>1; - 'J(1)
Luego, el valor de la pendiente en esta dirección es
244
Dü-f(6; 5) = 'í/[(6; 5) • Uií = f71 = 31.24
a v61
d) Para la dirección sur-oeste, se tiene e = 225 0
(Fig. 3.)$)

o .E
I ,lIl!gO, el vector unitario en la dirección sur-oeste es

u::;:: (cos 225°; sen 225°) ::;:: ( - V;; -V;)


Así, D,i!(6; 5) ::;:: ílf(6; 5) • u::;:: 22.J2 == 31,11
Por tanto, el hombre está subiendo con una rapidez de 31, II m/seg,
\ ) I I lugar geométrico de los puntos en la que el hombre debe recorrer alrededor
dl! la montaña manteniendo la misma altura que en el punto A(6; 5; 778),
(;orresponde a la curva de nivel
f(x; y) ::;:: 900 - 2x 2 - 2 y 2 ::;:: 778 ~ 2x 2 + 2y 2 ::;:: 122
<::::> X2 + y2 ::;:: 61 (circunferencia).

I J~mplo 24.- Calcule el valor de la derivada direccional de la función


1. [(x; y) ::;:: x 5 + xy + y3 en el punto A(l; 6), en la dirección de la curva
g(x) ::;:: 4X2 + 2
~fllllci6n

11 dl!rivada de la función 9 es~(x) ::;:: 8x


y
( ' 1'"10la curva y::;:: g(x) pasa por el punto
II ( 1; 6). entonces su dirección es dada por la
ll'll,1 tangente a la gráfica de 9 en A (Fig. 3.9).
1\ ,í la pendiente de la recta tangente a la c~
}' g(x) en el punto A es <~'"
mr ::;:: g'(l) ::;:: 8 o x

\ ~ u l!cuación es
Lr: y::;:: 8x - 2 F'g 39
1 lIl:go. la ecuación vectorial de la recra tangente es
Lr: (x; y) ::;:: (1; 6) + t(l; 8), t E ~
1 I \~' ctor unitario en la dirección de la recta Lr, esto es, en la dirección del vector
11 (1;8)es
_
Ha ::;::
a
lIall::;::
(1 8)
{65; {65
( ""lO íl {(x; y) ::;:: (5x 4 + y; X + 3y2), entonces el vector gradiente en el pUT\to
( I ,h) l!S
Vl(l; 6) ::;:: (11; 109)
1'111 1,11110, la derivada direccional de { en el punto A(l; 6), en la dirección de la

111\,1 y g(x) es
883
/JI; .[(1; 6) ::;:: íl{(1; 6).
n
ua
== r7'F
v65
Ejemplo 25.- Considere una función [(x; y), tal que
í/[(,x;y) = (4x 3 + 2xy4 + ye xy ; -3y2 + 4X2 y 3 + xe xy ) y [(O; O) = 21
La temperatura en un punto (x; y) de una placa rectangular con centro en el
origen está dada por
T(x; y) = [(x; y) + y3 - e XY
a) Determine la dirección en que una arafla debe ir, partiendo de] punto B( 1; 1) de
la placa, para que se enfríe lo más rápidamente posible.
b) ¿Cuál es la rapidez de la arafla en esta dirección?
Solución
Como [x(x; y) = 4x 3 + 2xy 4 + ye xy , entonces

[(x; y) = J(4x 3 + 2x y 4 + yexY)dx = x 4 + X2 y 4 + e XY + C(y)


donde cey) es una función de la variable y.
Al igualar las derivadas parciales de [ con respecto a y, se tiene
[y (x; y) == 4X2 y 3 + xe xy + C'(y) = -3 y 2 + 4X 2y 3 + xe xy
~ C'(y) = -3y 2 = C(y) = _y3 + k
Luego, ""
[(x; y) == x 4 + X2 y 4 ;;éY - y3 + k

Dado que [(O; O) = 21 ~ 1 + k = 21 ~ k = 20


Así, la temperatura de la placa rectangular es

T(x; y) = [(x; y) + y3 - e XY = x 4 +4/!y4 + 20


''l¡,.,
a) Como í/T(x; y) = (4x 3 + 2xy4; 4X 2y 3), entonces el vector gradiente de T en
el punto 8(1; 1) es
í/T(1; 1) == (6; 4)
Por tanto, la dirección que debe tomar la araña para enfriarse lo más rápido
posible es
v= -í/T(1; 1) = e-6; -4)
b) La rapidez de la araña en la dirección del vector es v
ID ü ¡¡T(l; 1)1 = 1-IIí/T(l; 1)111 = I-illl = ill

Ejemplo 26.- Sea [ex; y; z) == X2 + cos(x + y) - Z3 . Halle la derivada


direccional de [en el punto Po(l; -1; 1) en la dirección de un vector ortogonal a
la superficie de nivel de [ que pasa por Po
Solución
Como í/[(x; y; z) == e2x - sen ex + y); -sen (x + y); -3z 2), entonces el
vector ortogonal a la superficie de nivel de [ en el punto Po es
a = V[(l; -1; 1) = (2; O; -3)
d vector unitario en esta dirección es

1'01 tanto, el valor de la derivada direccional de [ en la dirección del vector


lIf togonal a su superficie de nivel en Po es
13
Da-[(1; -1; 1) = V[(l; -1; 1). ua = ¡;¡-;:; = ro
a v13

I Icmplo 27.- Sea [(x; y; z) = x2y2(2z + 1)2, Halle la derivada direccional de [


III el punto A(l; 1; -1), en la dirección de la recta tangente a la curva de
Illt\.:rsección de las superficies

SI: X2 + y2 + 2(y - x) - 2 = O

S2: x - y - 2z - 2 = O
dl' modo que al mirar la curva, desde el origen, el sentido es horario,
\ulución
1 \.:cuación cartesiana de la cu'~vade intersección de las superficies es
(x - 1)2 + (y + 1)2 = 4
e: {
y
x - y - 2z - 2 =O
1k donde las ecuaciones paramétricas del m~iento sobre la curva, en sentido
IHU ario está dado por ~~7
x-1
-2-= sent
y+1 X = 2 sen t + 1
e: -2- = cost ~ e: ¡zy==sen2 cost -t cos
- 1
t
x-y-2
z=----
2
1 .1 runción vectorial que describe el movimiento sobre la curva e es
e:a(t) = (2sent+1;2cost-1;sent-cost) y a(O) = (1;1;-1)
( ClIIIO a'(t) = (2 cos t; -2 sen t; cos t + sen t), entonces la dirección de la
l'll.1 tangente a la curva e en el punto a(O) = (1; 1; -1) es
ii = a'eO) = (2; O; 1)
1 l \cctor unitario en esta dirección viene dado por
a 2 1
li = Ilall = (.rs; O; .rs)
1'111 tallto, el valor de la derivada direccional de [ en el punto A, en la dirección
I 1\\,( tor IÍ es
D-Ua: [(1'1' -1)
J I
= V[(l'l''-1).11-
I a '
= (2' 2' -4). (~,
I .J'S' O'~)
'.J'S = O

Ejemplo 28.- Una partícula rastreadora de calor está situada en el punto A(S; 4)
de una placa metálica cuya temperatura en (x; y) es T(x;y) = 100 - X2 - 3y 2,
Halle la trayectoria de la partícula al moverse de forma continua en la dirección
de más rápido crecimiento de la temperatura,
Solución
Sea a(t) = (x(t);y(t)),t E 1 la función vectorial que describe la trayectoria de
la partícula en el plano XY,
Luego, el vector tangente en cada punto (x(t); y(t)) de la trayectoria es dado por
I (dX dY )
a (t) = dt; dt
Como la partícula busca el crecImIento más rápido de la temperatura, las
direcciones de a'(t) y VT(x;y) = (-2x; -6y) son iguales en cada punto de la
trayectoria, esto es
. dY) ._
a'Ct) = (dX
dt;dt = vt€x;y) = (-2x; -6y)
de donde se obtiene las ecuaciones

dX
dt
= -2x {dX
x
= -2dt
{ dy = -6y ~ dy = -6dt ~
dt y ~<Jo

Al integrar las dos últimas ecuaciones, se obtiene


flnx = -2t + el fx = e- 2t +C1 = e C1 , e-U fx = Ale-U
LIny = -6t + e2 ~ ty = e- 6t +C2 = é 2 , e- 6t ~ ty = A2 e- 6t
donde Al y A 2 son constantes reales, Así, la función vectorial que describe la
trayectoria de la partícula es
a(t) = (Ale-U; A 2 e- 6t ), tE 1 =- [O; +00)
Puesto que la partícula parte desde el punto A(S; 4), se tiene
aCO) = CAl; A 2 ) = CS; 4) ~ Al = 5 Y A 2 =4
Por tanto, la trayectoria de la partícula es la curva
4
e: aCt) = (Se-U; 4e- 6t ) ~ e: y = 12S x 3

Ejemplo 29.- Dada la función [Cx; y) = (2by - X)3. Calcule el valor de b para
que el valor de la derivada direccional máxima de [, en el punto A( -1; O) sea
igual a 3...[f7,
Solución
Como V[(x; y) = (-3(2by - X)2; 6b(2by - X)2), entonces el vector gradiente
de [en el punte A(·-1; O) es
V[(-1;0) = (-3;6b)
I,a derivada direccional de [ ·es máxima en la dirección dd vector gradiente
'iJ[( -1; O) = (-3; 6b) Y su valor es el módulo de este vector. e_'o es.
Düv[(-1; O) = IIV[(-l; 0)11 = ./9 + 36b 2 = 3m => b = ±2
Por tanto, los valores de b son b = - 2 ó b = 2

EJERCICIOS

I - Halle el gradiente de las siguientes funciones en el punto melicad('


a)[(x;y;z) =z 2e X seny Po(0;Tr/2;2 )

b) [(x; y; z) = ./X2 + y2 - z Po(2; -1; O)

c) [(x; y; z) = sen (3x)c~;x tan z PoCO; Tri 2; Tri 4)


d)[(x;y;z)=ln./x 2 +y2+ z 2 Po(-1;1;3)

e) [(x; y; z) = XZ + ZX + yZ + zY Po(2; 1; 1) R. (1; 1,3)

2.- Encuentrese la razón de cambio máxim


."q~e las siQlI
~
<:1l:CS funcIon es en el
punto que en cada caso se indica (IIV[II).
a) [(x; y; z) = xy2 + x 2z A(3;1; 2)
b)[(x;y;z) =excosy+eYsenz A(-1 ;2;2)
c) [(x; y; z) = (x + y)2 + Z2 - xy + 2z A( -2; .~; 2)
d) [(x; y; z) = XZ + ZX + yZ + zY A(4; 1;1)
e)[(x,y;z;t) =xz+y't A(l; O, -') .~)

1.- Calcule la derivada direccional de las siguientes funci ones en el punto P en la


dirección del vector PQ .

a) [(x; y) = e 2XY + 3x arctan(y) , P(O; O) y Q(l; -1)


bl[(x;y)=excosy+eYseny, P(1;0)y Q(-3 ;2)

c) [(x; y) = ./3x 2 - y2 - 2xy + 1, P(4; 3) Y Q(7; 7)

ti) [(x; y; z) = x sen (yz) , P(1; 1; O) Y Q(3; 2; -2)


cos(y sen z)
e) [(x; y; z) =
r
'xy
eCos tdt, P(O; 1; O) Y Q(1; 2; -1)

4.- Para cada una de las siguientes funciones, calcule el valor máximo de la
derivada direccional en los puntos que se indican, así como el vector dirección
en el que este ocurre.
x3 e x-t
a)[(x;y)=
x
+y ' P(1;3)

b) [(x; y) = cos(rrxy) - y sen (rrx 2 ), P(1; -1)

. x + In(y)
c)[(x;y;z) = , P(1;1;1)
z
?

d){(x;y;z) =.~ - xyz + yZz, P(-1; 1; O)


xy
fX 1
f
z
é dt +,;
2
e) [(x; y; z) = t2 dt , P(1; -1; 1)
x '>~ -y 1 +e

5.- Calcule la derivada direccional de la función [(x; y) = 2 y 3 x - 3X2 en el


. punto A(1; 2) en la dirección del vectorii = (A;-V1- ,F) (A > O).
Halle A para que esta derivada sea máxim ~. A = 5/13
.~\,

6.- Una función [ de dos variables tiene en el punto P(2; 3) los valores de las
derivadas direccionales de 4 en la dirección al punto A(3; 3) Y de -4 en la
dirección al punto 8(2; 4). Determine el vector gradiente de [ en el punto
(2; 3) Y calcule el valor de la derivada de [ en el punto P(2; 3) en la dirección
al punto Q(8; 11).
4
R. 'V[(2; 3) =-:: (4; -4) y Dü ,J(2; 3) = -5

7.- Sea [(x; y) = x2y. ¿Qué ángulo forma el vector dirección con la parte
positiva del eje X, si la derivada direccional en el punto P(1; -1) es 2?

R.e ~ arcsen (~)


8.- Calcule el valor de la derivada direccional de la función z = In(x + y), en la
dirección de la pendiente más pronunciada que caracteriza a la superficie
2z= In(e X + eY ) cuando z 1. =
e
R. Dfí[(X;Y) = r===
4 c
..Je - 2e

1) Si [(x; y) = J169 - x 2 - y2, encuentre el vector dirección U, tal que el valor


de la derivada direccional de [en el punto P(3; 4) es cero.
1
R. +-(4'-3)
-S '

I() ,', En que dirección [ex; y; z) = ex + y)2 + (y + z)~ + (z + x)~ creee mas
rápidamente en el puntO Po (2; -1; 2)? (,Cmii es la razón instantánea de
cambio de [ por unidad de distancia en esa dirección?
R. a=(-10;4;10)y IIV[(2;-1;1)11=..J216

11 - La altura de una colina sobre el nivel oel mar está dada por
h = [(x; y) = 200e- (H1) 2 + 80yc - ~Y :
donde x e y medidas en metros, son las coordenadas estc-()este y sur-nortc
res pectivamente. Un atleta ss. encuentra en el punto A(l ;O:ho)
,t) (,A qué altura se encuentra el t!cta 0
b) ¿En que dirección desde el punto A debe comenzar a caminar el atleta para
cscalar la colina lo más rápido posible? Si sigue esta dirección, ¿cuál es su
rapidez de cambio del atleta')
e) Si el atleta se mueve en la direcci'Ql~ sur-este, ¡,esta ascendiendo ()
descendiendo? ¡,Cuál es su rapidez? ~'?
d) Describa cllugar geométrico de los puntos que el atleta debe caminar. para
estar a la misma altura sobre el nivel del mar que en el punto 11.

11 La superficie de un lago se representa por una región D en el plano XY de


1110do que la profundidad debajo del punto (x; y) E D es dada por
[ex; y) = -10 - 2X 2 - 2 y 2
'-;i una pariguana se encuentra en el agua en ei punto (4;3):
a J ¿En que dirección debe nadar para que la profundidad debajo dc ella
di sminuya lo más rápido posible': ¿Cuál es el valor máximo de la derivada
direccional en esa dirección')
R. 6. = (-16;-12)y IIV[(3;4)1I = 20
h) (, En qué dirección debe nadar para que la profundidad debajo de ella
aumente lo más rápido posible: ¿,Cuál es el menor valor de la derivada
direccional en esa dirección?
1
l' ) ;, 1 n qué dirección no cambia la profundidad? ti = + r (- 3;-1)
, )
13.- Una nave espacial ha sobrepasado el planeta Marte cuando su capitán nota
que la cápsul" está comenzado a derretirse. La temperatura a su alrededor está
dada por T = 'x ; y; z) = 2e-(X-2) + 3e- 2(Y-2) + 4e 3 (Z-2)
Si la nave se encuentra en el punto A(2; 2; 2), ¿qué rumbo debe tomar para
enfriarse lo más rápidamente posible?

¡ 4.- Hay alguna dirección en la que la razón de cambio de


[(x; y) = 8x 2 - 2x2y - 7y2
('n el punto A(l; 1) sea igual a 217 (Justifique su respuesta?

15.- Halle la derivada direccional de la función [(x; y) = 4x 3 - 2xy + y4 en el


punto A (1; -1), en la dirección del vector que forma con la parte positiva del
eje X un ángulo de 30 0

¡ 6.- Considere que T(x) = 7 + 3x 2 + y2 representa la di stribución de la


temperatura en el plano XV (suponga que x e y se miden en metros y la
tem peratura en OC). Un h O~ll bre se encuentra en la posición A(l; 4) Y pretende
dar un paseo. ,.~;,.
al Describa el lugar geo métrico de los puntos que él debe recorrer si su deseo
es disfrutar sie mpre la misma temperatura que en el punto A.
R. 3x~ + y2 = 16
b) ¿Cuál es la dirección que debe tomar si ~deseo es caminar en el sentido de
mayo r asce nso de la temperatura?, ¿"rn.lál es la temperatura en esta
dirección')
R. Dirección ñ = (6; 8) Y 11\77'(1; 4)11 = 10
cl Si su deseo es caminar en la dirección del descenso má s rápido de la
temperatura, ¿q ué dirección debe tomar'~ R. Dirección E = (-6; -8)
d) Observe que el punto (O; O) es el punto más frío del plano Xl'. Encuentre la
trayectoria que el hombre (que busca el frio) debe seguir haci a el origen,
partiendo del punto A( ¡ ;4) R. Y = 4 VX
e) ¿En qué dirección debe moverse desde A( 1;4) si su deseo es que la
temperatura aumente a razón de 4°C/m7

¡ 7.- La altura del volcán Sara Sara. en metros sob re el ni vel del mar. está dada por
2 y2
h = 5400 - x --
2
Si un alpinista comienza su ascenso en el punto A(40;20), ¿cuál es la
trayectoria en el plano XV que corresponde a la nna más empinada de ascenso
al volcán" R. )l = V10x
1R.- La temperatura en un punto (x; y; 7.) de un sólido está dada por
T(x; y; z) = cos(xy) + eX"+Y"+z~ - III(y7.)
a) Calcule la razón de cambio de la temperatura en el punto peO; 1; 1) yen la
a e
dinxciólI del vector = -1; 2; 2).
11) ¡,[n qué dirección T crece más rápidamcnte? ¿,A qué ritmo'.'

I().-Dada la ("unción 9(X; y) = e X +Y + IX ~ dt


o t +1 4

a) Calcule la derivada direccional de 9 en el punto (O; O) en la direcciún del


a
vector = (2; 1)
b) ¿En qué dirección ia derivada direccional de 9 en (O; O) toma el valor
máximo'.'

'().- ¿C uál es la razón de cambio de la ("unción {(x; y; 7.) = x~ + y2 + z - 4 a lo


. (!' cos A 1 3 3 3 )
largo de la curva a(8) = \,. 2 ; '2 + '2 sen (); '2 - '2 sen e en el punto
Tf ";¡;,

que corresponde a () = -6 7.

) 1.- Ca lcule las constantes (1, b Y e para que la derivada direccional de la ("unción
f'(x; y; z) = axz 2 + bxy + cx:y~ en el ~ to P(l; 1; -1) tenga un valor
múximo de 4, y esté en el sentido positivo dcleje Z.
R. a = ±2, b = +2, e = ±1

,) Sea z = [(ax + by). donde a y b son constantes positivas y [es una función
derivable.
,1) Demuestre que en cualquier punto A(xo; Yo), el vector gradiente de 7. es
paralelo al vector ti = (a; b)
h) J)etermine los punto Q(x 1 ; Yl) tales que la derivada direccional de z en Q y
cn la dirección del vector v = (-b; a) es igual a cero.

x It-l)Z

d)
l)ada la función [(x; y) =
I
1 e---:1' dt - (2y - X)2

II.llle la derivada direccional de f en el punto A(l; -1) segLIIl la dirección


11m-este . En esta dirección, ¿a umenta o disminuye el valor de F'
11) Determine la dirección del descenso más rápido de [ en el punto A. ¿,Cuúl
~.~ el valor de la derivada direccional de f en esta dirección?
24.- Un grupo de alpinistas escalan una montaña que se encuentra sobre la
Cordillera Blanca ubicada en el departamento de Ancash. Suponga que la
altura de la montai'ía sobre el nivel del mar viene dada por la ecuación
z = 6 - 2X2 - 3xy + y3 (las distancias ' se miden en kilómetros) . Los
alpinistas se encuentran en el punto A (1; 2; 6) a las 12 de la noche en plena
oscuridad.
a) Los alpinistas no se ponen de acucrdo qué dirección deben seguir para
escalar lo más rápido posible a la cima de la montai'ía, por lo que deciden
calcular la pendiente de la monmi'ía en el punto A en la dirección norte y en
¡a dirección nor-oeste. Si deben seguir por la ruta de mayor pendiente,
¿c uál de las dos direcciones deben elegir? R. Deben elegir la dirccción
nor-oeste.
b) ¿Cuál es la dirección de máxima pendiente en A? ¿Cuál es el valor de dicha
pendiente') R. Dirección a = (-10; 9) Y IIV[(l; 2)11 =.JTIIT
25.- Sea [(x; y; z) = in(x 2 ty2 + Z2). Halle la derivada direccional de J c:n el
punto (1; 3; 2) a lo larg;";uc' la curva de intersección de las superficies
Si: 36x2 + 4y~ + 9z 2 = 108 Y S2: X2 + y2 - 5z = O, si al mirar éste desde
el origen, el sentido es horario.
36
~
.. R. DüfO; 3; 2) = ~
.,. IV 194

26.- Sea e la curva de intersecc ión de los cilindros x~ + y2 = 1 Y x 2 + z~ = 1.


en el primer octante. Ilalle la derivada direccional de la función
[(x; y; z) = X2 + y2 + Z2 a lo largo de esta curva en el punto

2..[6
R. Dü[ = -~-
.)

3.4 PLANO TANGENTE Y RECTA NORMAL A UNA SUPERFICIE

Teorema 5.- Sea S: Fex; y; z) = O la ecuación de una superficie, donde


, F(x; y; z) es una función con primeras derivadas parciales continuas. Si ¡'~, F~
Y Fz no son todos ceros en el punto Po(xo; Yo; zo) E S, entonces el vector
Ñ = VF(x o; Yo; zo) es normal al plano tangente a la superfic ie S en Po.
Dcfinición 8.- Sea S: F(x; y; z) = O la ecuación de una superficie, donde
F(x; y; z) es una función con primeras derivadas parciales continuas en
!'o(xo;Yo;zo), con VF(xo;yo;zo)"* O,

i) El plano que pasa por Po Y es normal a VF(Po) se denomina plano tangente a S


en Po Y tiene por ecuación general

ii) La recta que pasa por Po Y tiene la dirección de VF(xo; Yo; zo) se denomina
recta normal a S en Po Y tiene por ecuación vectorial

La ecuación sil1lé~rica de la recta normal a S en Po esta dada por

Ejc mplo 30.- Halle las ecuaci6nll5 del plano tangente y de la recta normal a la
superficie 4x 2 + y2 - 16z = O en el punto (2; 4; 2),
Solución
:\1 considerar F(x; y; z) = 4x 2 + y2 - 16z = O, se ticnc

VF(x;y;z) = (8x;2y;-16)

Asi, el vector gradiente en el punto Po(2; 4; 2) es

VF(2; 4; 2) = (16; 8; -16)

Luego, la ecuación del plano tangente en Po(2; 4; 2) es

P.,.: 16(x - 2) + B(y - 4) - 16(z - 2) =O~ P.,.: 2x +y - 2z - 4 = ()


La ecuación simétrica de la recta normal es
x-2 y-4 z-2
LN'' -2- -
- -1- -
---2-

Observac ión 7.- Sea e la curva de intersección de las superficies


F(x;y;z) = O Y C(x;y;z) = O
l a recta tangente a la curva e en el punto Polxo; Yo; zo). es la recta intersección
de ios planos tangentes a las superficies F(x; y; z) =O J C(x; y; z) =O
en el punto Po. Luego, los vectores normales Ñ1 = 'ilF(xo; Yo; zo) y
Ñ2 = 'il C (x o; Yo; zo) son ortogonales al vector tangente a la curva e en Po.
e
Por tanto, el vector tangente a la curva en el punto Po tiene la misma dirección
que el vector Ñ1 X Ñ2 .

Observación 8.- Si la ecuación de una superficie está definida de manera


explícita por z = [(x; y), se define la función F por F(x; y; z) = [(x; y) - z = O
Y la ecuación del plano tangente en el punto Po (xo; Yo; zo) viene dada por

Ejemplo 31.- Halle la ecuación vectorial de la recta tangente a la curva de


intersección de las superficies e: X2 + y2 - Z = S Y x - y2 + Z2 = -2 en el
punto Po(Z; -2; O).
Solución
Sean las superficies:
F(x; y; z) = X2 + y2 - Z - S Y C(x; y; z) = x - y2 + Z2 + 2
Luego, los vectores gradientes <~.e tas funciones son

'ilF(x;y;z) = (2x;2y;-1) y 'ilC(x;y;z) = (1;-2y;2z)


En el punto Po(2; -2; O). los vectores normales de los planos tangentes son

Ñ1 = 'ilF(2; -2; O) = (4; -4; 1) Y Ñ2 = 'ilCetr-2; O) = (1; 4; O)


.~~

De donde se obtiene a= Ñ 1 X Ñ2 = (4; -1; 20)


Por tanto, la ecuación vectorial de la recta tangente a la curva e en el punto Po es
LT : (x; y; z) = (2; -2; O) + te 4; -1; 20). t E lRt

Ejemplo 32.- Halle el valor de m para el cual el plano Q: x - 2y - 2z +m =O


es tangente a ·la superficie S: X2 + 4y2 + 16z 2 = 144
Solución
Al expresar la ecuación de la superficie S en su forma explícita, se tiene
F(x; y; z) = X2 + 4y2 + 16z 2 - 144
Si Po (xo; Yo; zo) es el punto de tangencia del plano tangente, entonces su normal
es
Ñ = 'ilF(xo; Yo; zo) = (2xo; SYo; 32zo)
Como el vector normal del plano dado ÑQ = (1; -2; -2) Y Ñ son paralelos. se
sigue que
Ñ x ÑQ = (-16yo + 64z o; 4xo + 32zo; -4xo - sYo) = (O; O; O)
I )e tlonde resulta Xo = -8zo y Yo = 4z o
Pucsto que Po( -8zo; 4zo; zo) es un punto de la superficic S, entonces sus
l'oordenadas satisfacen su ecuación, esto es

64z~ + 64z~ + 16z~ = 144 ~ Zo = ±1

1.1Iego, los puntos de tangencia son

Po(-8;4;1) y P' o(8;-4;-1)


Po r tanto, al ser Po (-8; 4; 1) Y p' 0(8; -4; -1) puntos del plano
(¿: x - 2y - 2z + m = 0, se obtienen m = 18 para el punto Po Y m = -18 para
e l punto P'o.

X2 Z2
Ejemplo 33.- Halle los puntos de la superficie s: 4 + y2 + 4 = 11, en los
cua les el plano tangente a S es paralelo al plano Q: x + 2y + 3z = 3. Para cada
llIlO de los puntos obtenidos, escriba la ecuación general del plano tangente .
";o lución
Cons ideremos la ecuación de i5<w,Perficie como
X2 Z2
F(x;y;z) = 4+ y2 +4- 11

Como 'VF(x; y; z) = (~2; 2Y;:),


2
entonces el vtf!tor normal del plano tangente
.~
7
a la superficie S en el punto Po (xo; Yo; zo) es

Dado que el plano tangente Pr y el plano Q' son paralelos, se sigue que sus
vectores normales Ñ y ÑQ = (1; 2; 3) son paralelos, lo cual significa que

De donde resulta: Xo = 2yo Y Zo = 6yo


Puesto que P(2yo; Yo; 6yo) es un punto de la superficie de S, sus coordenadas
satisfacen su ecuación, es decir

S: y~ + y~ + 9y~ = 11 ~ Yo = ±1

Luego. los puntos de tangencia son: P1 (2; 1; 6) Y P2 (-2; -1; -6)


Po r tanto, las ecuaciones generales de los planos tangentes en los pumos PI y I)~
son respectivamente
QI: x + 2y + 3z - 22 = O Y Qz: x + 2y + 3z + 22 = O

Ejemplo 34.- Sea e la curva de intersección del paraboloide z = 9- XZ - yZ con


el plano x = l.
a) Halle la ecuación vectorial de la recta tangente a la curva e
en el punto
PI (1; 2; 4).
b) Halle la ecuación del plano tangente a la superficie s: 4x 2 + 3y2 - 24z = O,
que es perpendicular a la recta tangente obtenida en a).
Solución
La función vectorial que indica la posición de un punto sobre la curva es e
C: a(t) = (1; t; 8 - t Z )
Para t = 2, se obtiene a(2) = (1; 2; 4)
Como a'(t) = (O; 1; -2t), entonces a'(2) = (O; 1; -4)
e
a) La ecuación vectorial de la recta tangente a la curva en el punto PI (1; 2; 4).
que sigue la dirección del vector a'(2) es

LT : (x; y; z) = (1; 2; 4).• + t(O; 1; -4), t E lR1.


"'.
b) Sean F(x; y; z) = 4X2 + 3y2 - 24z Y P(xo; Yo; zo) el punto de tangencia del
plano tangente a la superficie S. Luego, se tiene

Ñ = 'ilF(Po) = (8xo; 6yo; -24)


Como el plano tangente es perpendicular-<ia
q~.
recta tangente obtenida en aJ.
entonces el vector normal Ñ y el vector dirección de la recta tangente
a = (O; 1; -4) son paralelos, lo cual implica que
Ñx a = (-24yo + 24; 32xo; 8xo) = (O; O; O)
De donde resulta, Yo = 1, Xo = O
Así, en virtud de que P(O; 1; zo) es 1111 punto de la superficie S, se tiene
1
0+ 3 - 24z o = O ~ Zo = 8"

Por consiguiente, la ecuación del plano tangente que pasa por P (O; 1;~) es
PT :2y-8z-1=0

Ejemplo 35.- Demuestre que la suma de los cuadrados de lasjntersecciones con


los ejes coordenados de cualquier plano tangente a la superficie
X2/ 3 + y2/3 + Z2/3 = b 2/ 3 es constante e igual a b 2.
Solución
Sean. F(x; y; z) = X2/ 3 + y2/3 + Z2/3 - b 2/ 3 Y Po (x o; Yo; zo) el punto de
tangencia de la superficie. Entonces

Ñ = ílF(P)
o
= (~3 Xo-1/3.~
' 3 Yo
-1/3.~ -1/3)
' 3 Zo

Luego, la ecuación del plano tangente a la superficie en Po es

PT : Xo-1/3 ( x - Xo ) + Yo-1/3 (y - Yo ) + Zo-1/3 ( Z - Zo ) = O

el cual es equivalente a

PT : ~/3 + ~/3 + :/3 = b 2/ 3 (x~/3 + y~/3 + z~/3 = b 2/ 3)


Xo Yo Zo
Las intersecciones del plano tangente con los ejes X,Y y Z son respectivamente

x = x~/3b2/3,y = y~/3b2/3 Y Z = Z~/3b2/3

Por consiguiente, la suma de los cuadrados de estas coordenadas es


~1><

X2 + y2 + Z2 -- (X o2/ 3 ~'v.U3
JO + Z2/3)b
o
4/ 3 -
- b 2/ 3b 4 / 3 -- b 2

EJERCICIO .

1.- Determine la ecuación general del plano tafígente y de la recta normal, para
cada una de las superficies, cuyas ecuaciones se dan a continuación, en el
punto P dado.
a) X2 + y2 + Z2 = 17, P(2; -2; 2) R. 2x - 2y + 3z = 17
x
b) z = - - P(2' -1' 2)
x+y' "
c) XS + yS + ZS = 30 - xyz, P(2; 1; -1)
d) X1/2 + y1/2 + Z1/2 = 4, P( 4; 1; 1)
e) 2x 2 - xy2 - yz2 = 18, P(O; -2; 3) .
f)x 2 / 3 +y2/3 +Z2/3 = 14, P(-8;27;1)

2 - En los siguientes casos, halle la ecuación vectorial de la recta tangente a la


curva de intersección entre las superficies dadas en el punto indicado.
n)y=x 2 ,y=16-z 2 , P(4;16;0)
b) X2 + Z2 + 4y = 0,x 2 + y2 + Z2 + 7 = O, P(O; -1; 2)
c) x = 2 + cos(1l'Yz) ,y = 1 + sen (1l'xz), P(3; 1; 2)
d} xyz = 36, 4x 2 + y2 - 2xz 2 = 105, P(6; 3; 2)
3.- Dada la función [(x; y; z) = arcsen (
X
2
3y2
6 + -2- + 24 - '2
Z2 1)
a) Halle la ecuación del plano tangente a la supertlcie de nivel

[(x; y; z) = i, en el punto (1; ~; -4) R. x + 3y - z = 6

b) ¿En qué proporción varian los valores funcionales cuando comienza a


moverse desde el punto (1; 1/3; -4) hacia el punto (2; -5/2; -2)?
Sugerencia: Aplique el concepto de la derivada direccional.

X2 y2 Z2
4.- ¿En qué puntos del elipsoide a 2 + "b2 + c 2 = 1 la normal forma ángulos
iguales con los ejes coordenados?

5.- Dada la superficie X2 + 2y 2 + 3z 2 = 21. trace a ella planos tangentes que


sean paralelos al plano x + 4y + 6z = O

6.- Halle la ecuación del plano tangente a la superficie 4y2 - 2X2 - 7z = O que
x z
pase por el punto (-8; O; 4) Y sea perpendicular al plano ¡ -7 = 1
R. 4x ± 4.J6y + 7z + 4 = O
·,ó
7.- ¿En qué puntos de la gráfica de la superficie S: 4x 2 + y2 + Z2 - 2xy = 12.
los planos tangentes a la superficie son paralelos al plano YZ0
R. (2; 2; O) Y (-2; -2; O)

8.- Halle las ecuaciones del plano tangente y recta normal, si se sabe que el plano
tangente es hOrizontal a la gráfica de la superficie z = XZ + 4 y z + 1
R. Plano tangente: z= 1

9.- Verifique que la suma de las intersecciones con los ejes coordenados de todo
plano tangente a la superficie yX + /Y + .¡z = Va, a > O es constante e
igual al valor de a.

10.- Ilalle la ecuación del plano tangente a la superficie X2 - y2 - 3z = O que


JX - 2y = O
pase por el punto (O: O: -1) y sea paralelo a la recta t2y _ z = O
R. 4x - 2y - 3z = 3
11.- Halle en la superficie X2 + y2 - Z2 - 2x = O los puntos en que los planos
tangentes a ella sean paralelos a .los planos coordenados. R. En los puntos
(1; ±1; O) los planos tangentes son paralelos al plano XZ, y en los puntos
(6; O; O) Y (2; O; O) al plano YZ. La superficie carece de puntos en los cuales
el plano tangente sea paralelo al plano XY.

12.- Halle la mayor razón de cambio de la función

[(x; y; z) =e x2
cos((x + 2y) rr) + 16 y 2 + Z2
en el punto Po, donde Po es un punto de la superficie
s: Z2 = X2 - 2y 2 + 3y - 6
en el que el plano tangente es paralelo al plano 2x - y - 2z = 6

X2
13.- Halle la ecuación del plano tangente a la superficie 2" + y2 + 7z 2 = 126

que es ortogonal a la recta tangente en (2; 1; 6) a la curva de intersección de


las superficies z = X2 + 2y~¡,! Z = 2x 2 - 3y2 + 1

R. 5x + 2y + 28z ± 166
í63 = O
183
~

14.- Halle el valor de k para que en todo p(to de ia intersección de las dos
esferas ex - k)2 + y2 + Z2 = 4 Y X2 +"(y - 1)2 + Z2 = 1, los planos
tangentes sean perpendiculares uno al otro. R. k = ±2

15 .- Def. Dos superficies son tangentes en el punto p. si tienen el mismo plano


tangente en P. Demuestre que las superficies dadas son tangentes en P.
a) X2 + y2 + Z2 = 2, yz = 1, peO; 1; 1)
b) X2 + y2 + Z2 = 3, xyz = 1, pe1; 1; 1)

1(¡ - Def. Dos superficies son ortogonales en el punto P si sus normales en P son
perpendiculares. Demuestre que las superficies dadas son perpendiculares
l: lltre si en el punto dado.
a) 3X2 + 2y 2 - 2z = 1, X2 + y2 + Z2 - 4y - 2z + 2 = O, (1; 1; 2)
b) X2 - y2 + Z2 = -2, X2 + y2 + 3z 2 = 8, (-1; 2; 1)
3.5 INCREMENTO Y DIFERENCIAL DE UNA FUNCIÓN DE VARIAS
VARIABLES

Para una función [ de varias variables no es suficiente la existencia de las


derivadas parciales en un punto para decir que la función es diferenciable en ese
punto, puesto que la existencia de la derivada en la dirección de los ejes
coordenados no implica la existencia de la derivada en otras direcciones.
Es más, aunque la derivada de la función existiera en un punto en todas las
direcciones, no se puede garantizar la existencia del plano tangente a la gráfica de
{ en ese pUllto.
Por ejemplo, una función [ de dos variables, geométricamente es diferenciable en
un punto (xo; Yo) E Dr si existe plano tangente a la gráfica de [ en el punto
Q(xo; Yo; [(xo; Yo)); Y esto ocurre cuando la gráfica de f es suave en ese punto.
es decir, su gráfica no tiene vértices. puntos criticas, etc. en ese punto.
La diferencial de una funcil"1l de varias variables es el concepto teórico que
garantiza la existencia del planb--~tangente a la gráfica de la función en un punto.
Vamos a empezar a definir el concepto de incremento de una función de varias
variables. Luego, se presenta el concepto de diferencial de una función de varias
variables.

Definición 9.- Sea [: D e JRl.2 -+ JRI. una función de dos variables, tal que
z = [(x; y), y sean tu y !:.y los incrementos de x y de Y respectivamente. El
incremento de z = {(x; y) en cualquier punto (x; y) E D es dado por

!:.f(x; y) = f:,z = {(x + !:.x; Y + !:.y) - [(x; y)


En general se tiene la siguiente definición:

Definición 10.- Sea [: D e JRl.n ..... JRI. una función de n variables, tal que
z = [(Xl; ... ; x n ) y sea !:.P = (!:.x l ; ... ; !:.x n ) el incremento del punto
P(x l ; ... ; x n ) E D. El incremento total de la función [ o simplemente incremento

de [ en P es dado por
!:.[(P) = f:,z = [(Xl + !:.xl; ... ; x n + !:.x n ) - [(Xl; ... ; x n ) ó
!:.[(P) = [(P + !:.P) - [(P)
Ejemplo 36.- Sea [(x; y) = 3x 2 + 2xy - y2, tu = 0,03,!:.y = -0,02
lI alle el incremento de [ en el punto P(l; 4).
Solución
1:1 incremento de la función z = [(x; y) en el punto P(l; 4) es
!:.f(1; 4) = [(1 + 0,03; 4 - 0,02) - [(1; 4) = [(1.03; 3,98)
= [3(1.03)2 + 2(1,03)(3,98) - (3,98)2] - [3 + 8 - 16] = 0,5411
Definición 11.- (Diferenciabilidad de una función) Sea [: D e 1Ri~ -) lRi una
fu nción de dos variables, tal que z = [(x; y) , y sea !:.P = (!:.x; !:.y) el incremento
de l punto P(x;y) E D.
Se dice que la función [ es diferenciable en P(x; y), si existe el vecto r gradiente
íl [e p) tal que para P + !:.P E D el incremento !:. Z se puede expresar en la forma

~f(x ', y} = ~z = O.J(x·, y}~Y. -+ 0 "2 f (1:', y)6.y -+ c"6.x -+ c¿6.y Ó

[ (P + !:.P) = [(P) + íl[(P) ·!:.P + Ee!:.p),

E.i emplo 37.- Sea [(x; y) = 4x - 2xy 2. Demuestre que [ es diferenciable en


todos los puntos de 1Ri 2 .
,",o lución
Para P(x; y) E 1Ri 2 Y !:.P = (!:.x; !:.y), se tien~
<~

lep + !:.P) + !:.x; Y + !:.y) = 4(x + !:.x) ..::: 2(x + !:.x)(y + !:.y )2
= [(x
= 4x - 2xy 2 + (4 - 2y 2)!:.X - 4xy!:.y - e4y!:.y)!:.x - [2x!:.y + 2!:.x!:.y]!:.y
= [(x ; y) + íl[(P) • ílP + E(!:.X; !:.y),
donde
lim
fI ' .lIy)->(O; O)
Ee!:.X; !:.y) = lim
(lIx :lIy) -> (o :O)
[-(4y!:.y)!:.x - (2x!:.y + 2!:.x!:.y)!:.y] = °
POI consiguiente, [ es diferenciabl e en cualquier punto (x; y) E 1Ri 2 . En general,
l' tiene la siguiente definición:

Ill'finición 12.- Sea f: D e 1Rir. ..... lRi una función de n variables tal que
{(Xl ; ... ; x ll ) Y sea !:.P = (!:.x 1 ; ••• ; !:.x ll ) el incremento del punto
1'(I' I;"' ; X Il ) E D.
,1 dile que f es diferenciable en P(x]; ... ; x si existe el vector gradiente íl (U»,
ll )

I,d 'lI le para p + !:.P E D el valor de la función se puede expresar en la forl1la


fep + !:::.P) = fep) + Vfep) • VP + Ee!:::.p)
donde lin1...Ee!:::.P) = lim Ee!:::.X1 ; oo.; !:::.x n ) = O
llP .... O (llXl;···;llxl1) .... (O; ... ;O)

Teorema 6.- Si una función f: D e 1R{2 -t IR{ de dos variables es diferenciable en


el punto poexo; Yo) . entonces f es continua en Po·

El recíproco del Teorema 6 no es necesariamente verdadero, como se puede


observar en el siguiente ejemplo.

Ejemplo 38.- Demuestre que la función fex;y) = ~X2 + y2 es continua en


[O: O,1,nero no es rli(erJ~t1Úahle.?ll!.n·.n)

Solución
i) feO; O) =o ii) lim fex;y) = (X;y)lim ~X2 + y2 = o
lx;y) .... (O;O) .... (O;O)

iii) lim fex; y)


(x;y) .... ( o ; O ) · ¡ .
= f(Q:, O) = o
Por tanto, f es continua en (O: O). Por otro lad o,

Vf(x'Y) = (' x . Y ')


, ~ X2 + y2 ' ~ X2 + y2 ..,

no existe en (O: O). Luego, f no es diferenciable"'~11 (O; O),

Teorema 7.- Si la función f: D e 1R{2 --> IR{ y las funciones


8{(x;y) 8{(x:y).
fAx; y) = y fy (x: y) = 8 son continuas en D, entonces
8x Y
fes diferenciable en D,

xCy2 .
·
EJemp o .- ea fe x: Y) =
l 39 S (2
x +Y 2)2 ,Si (x; y) ;¡= (O; O)
{
O , si ex; y) = (0:0)
Demuestre que fAO; O) y fy(O: O) existen, pero que f no es diferenciable en
(0;0),
Solución: Se tiene
f(h:O)-feO;O) O-O
{y(O; O) = limIl~U
,
11
= hIim
.... O
- .- = O
11
f(O;k)-f(O;O) o-o
Fy (O; O) = lim
k-O k
= lim - - = o
k~O k
luego, las derivadas parciales de f en (O; O) existen.
'\hora, sean TI={(X;y)ElR1.~/y=x} y T2 = {(x;y) E lR1.¿/y= Oj dos
trayectorias que pasan por el punto (O; O). Los límites de f sobre estas trayectorias

1
Sobre TI: lim f(x;y)=limf(x;x)=-
(x;y)-(O;O) x-->O 4
Sobre T2 : lim
(x;y)-(O;O)
f(x; y) = x-o
lim f(x; O) = O

Asi, por la regla de las dos trayectorias lim f(x; y) no existe. lo que
\X;y)~(O;OJ

significa que f no es continua en (O; O).


Por consiguiente, en virtud del teorema 7, f no es diferenciable en (O; O).

Definición 13:- (Diferencial Total). Sea f: D e lR1. z ...... lR1. una función de dos
variables para la cual existen f~x; y) y fy(x; y) en todo (x; y) E D, Y sean tu y
t:.y incrementos de x e y respectivamente.
i) Las diferenciales de las variables independientes x e y son
dx = t:.x • dy = t:.y
ii) La diferencial total de f. denotada por ~; y) es

df(x; y) = af~; y) dx + af~x; y) dy '1\,


x y
La extensión de esta definición a funciones de n variables es el siguiente:

Definición 14.- Sea f: D e ffi¡," ...... lR1. una función de n variables para la cual
I:x.isten DJ(x l ; X2; ... ; x ll ) para i = 1.2 ..... n y sean L\x l • ... . t:.x n incrementos de
las variables Xl' .... x ll respectivamente.
i) Las diferenciales de las variables independientes Xl' ...• xr¡ son

dX I = t:.x l . dX2 = t:.x 2• ...• dX ll = t:.x ll


ii) La diferencial total de f, denotada por df (Xl' .... x n ) es
a(x l • ...• x ll ) a(x1 • ..• x n )
df(xl.···.x n ) = a dx 1 + .. ·+ a dX ll
Xl XI!

¡':¡cmplo 40.- Halle la diferencial tot;;¡1 eJe la función


z = f(x; y) = ~
x3
+ arctan (\x~)
3
Solución
Las derivadas parciales de f son
3y 3yx2
fx(x;y)=--¡- ó+ 2'
x X y
Luego, la diferencial total de fes
2 i
3y 3yx \ ( 1 x: )
d7.=df(x;y)= ( - - 4 - ( ")dx+ --:;-+ . " ely
x x' + Y' X·' xl> + Y"

Observación 9.- Sea f: D e 1R{" ...... IR{ una función difcrenciable en el punto
P(x}; ... ; XI!) E D, es decir

!::.f(P) = D¡J(P)!::.x 1 + ... + D,J(P)!::.x" + E(!::.P)


donde lim E(!::.P) = O
b? - /ó

Asi, cuando!::.P = (!::.x}; ... ; !::.Xl!) ...... (O; ... ; O), el cambio real de
z = f(x}; .oo; XI!) es aproximadamente igual a la diferencial total dz, es decir.
t:.z = !::.f(P) == dz = cPHP
De donde resulta
f(P + !::.P) == f(P) + df(P)

Ejemplo 41.- Use diferenciales para c lar el valor aproximado de


V6(1,98)3 + (4,1)2 '~"
Solución
Al considerar la función f (x; y) = V6x 3 + y2, se puede calcular con facilidad
f(2;4) = \/48 + 16 = Zt64 = 4
Luego. al tomar Xo = 2, Yo = 4, dx = !::.x = -0,02 Y dy = !::.y = 0,1 , se tiene

flx o + t:.x; Yo + !::.y) = fO,98; 4,1) = .16(1,98)2 + (4,1)2


== f(2; 4) + df(2; 4)
== 4 + fA2; 4)dx + f y (2; 4)dy
3
6x 2)'
Como fAx; v) = (6x 3 + y2)2/3 Y fy(x; y) = 3(6x 3 + y2)2/3' entonces
se tiene
48 8 1
fx(2; 4) = 16 = 3 Y fy (2; 4) = 48 = (;
Por tanto,
1
~ 6(1,98)2 + (4,l)2 == 4 + 3(-0,02) + ;- (0,1) = 3,9567
o

PROPA GACiÓN DE ERRORES

Si :: y denotan los valores medidos de dos var iabl es y x + 6x y y + 6y


~
::::p re ~(: ntan
Ics",aiorcs exactos, entonccs !1x y 6y son los errores de mcdición.
Si ¡os valores medidos de x e y se usan para calculm ei valor de la variable
dependiente z = [(x; y), el1lonces el error prupagado en la med ida de la variable
7. es

M = [(x + 6x;y + 6y) - [(x;y)

Observación 10.-
a ) El error propagado en la variable z = f(x;y) ,>c e\tilll,l (l se aprO';ima por la
dit'crencial total de z. esto es
6z == elz = I~(x; y)clx + f~(x; y)cly
b) Para determinar si el elT0r propagado en la variable dependiente z es grande o
pequei'io, se usa el error r<t.~lti\'o Y el error porcentual de z = {ex; y) en el
PUllto (x; y). Así, se tiene:

i) El error relativo o cambio relativo de {en I'(x; y) se est imil por


l:' . R. (t(P)) = f:jf(P) == d[(P)
[(1") l(P) -.1'
ii) El error porcclltual de [ expresa el cambfu de { CO IllO un porcentélje de su
tamai'io antcs del cambio y se estima por
E. P. (t(P)).= M(?) x 100 == d[(P) x 100
[(P) f(P)

".jcmplfl ·n.- El radll' de la base) la allUra de UIl COIlO circular recto miden ~() CIll
\ 5U cm rl':, pectivamcllte, con un posible error en Id medición de 0,0 I Utilice CI11.
dilc renciales para estimar el crror ma"imo y el error porcelltual cn el calc ulo del
\olu lllcn del COIlU.
,",olución
1
I I \olumen del cono es V = -;:;rr)'zh, con
.)
lo cual su dil'crencial total es
av é)V 2rrrh rr)'~
dI' = -- e[¡' + - cih = - - el)' +- dI¡
a)' ah 3 ~)

( ' IIIIlO), = 20, h = 50, el)' = 6)' = 0,01 Y dh = 6.h = 0,01. entonces

2000n 400rr
dI' = -.,- (0,01) + -.,- (0,01)
) . ~I
= Hn
Por tamo, el máximo error en el cálculo del volumen es aproximadamente 8rr cm.
El error porcemual estimado en el cálculo del volumen es
dV 8rr
E. P. (V) = V x 100 = 20000rr (100) = 0,12%
3

Ejemplo ·B.- El error porcentual cometido en la medición dc la altura de una


torre es 1% , para lo cual se mide el ángulo dc
Torre
elevación hasta la parte superior desde el punto A
sit uado a . +o m de su base. Ei ángulo mcdido es de
45° con un posible error de ±O,5 minutos.
Suponiendo que el terreno és plano, halle en forma
aproxilll.ada el máximo error porcentual cometido en
la mcdición de la distancia dcl punto A a la base de A" - I . - - - - -
x
la torre.
Solución
Si x es la distancia del punto A a la base de la torre. e el ángulo de elevación y h
la altura dc la torre, entonces d~•..Ia figura 3. lOse tiene
h = x tan e
Lucgo. la diferencial total de la altura es
ah ah
dh = -a dx + -a de = tan e dx + x sec 2 e de
x e ~
De donde se obtiene
dh tan e x sec 2 e
h = -¡;-dx + --¡¡-de
Ahora, dc la relación de medidas entre grado y mlJluto se obtienc

JI ° ..-, 60min} = z. = (_1_)' " Yf'180° -; "


1T rad } 1T
lz ..-, 0,5 min 120 l'
l (120) -. y rad
=y=
(120)(180)
HU!.

Lucgo, al reemplazar los datos


1T
X = '~O, e = 45°, de = 0,5 min = (120)(180) = 0,000145 rad
dh 1
h = 40 tan 45° = 40 v -
J h
= --
100
= 0,01 en (*), se tiene
2
tan 45° (40 sec (45 0))
0,01 = 40 dx + 40 (0,000145) = dx = 0,3884
Por tanto. el crror porcentual cometido en la medición de la distancia del punto A
a la base de la torre es
dx (0,3884)
E. P. ex) == - (100) = . (100) = 0,971 %
x 1. 40
Ejemplo ·U.- Si el radio y la altura de un cilindro aumentan en 0,5% y 1%
respectiva mente, ¿cuál es el cambio porccntual aproximado en su volumen?
So lución
El volumen del cilindro de radio r )' altura h es
V =: rr1'~h
De donde resulta
av av .,
dV =: - el1' + - elh =: 2rrrhelr + rrr" elh
81' ah
el1' elh
Como E. P. (1') =: - x 100 =: 0,5 Y E. P (h) =: -h x 100 =: 1, el cambio
r

porcentual aproximado en el volumen del cono es

elV
E. P. (V) == - x 100 =:
el1'
2 - x 100
elh
+ -1 x 100 =: 2 -
(1) + 1 =: 2%1
V r 1 2

Ejem plo ~5.-La resistencia total R de n resistencias U ¡, R2' . .. , Un conccradas en


paralcl~ está ldada or i 1 '\. ...-~~
-=-+-+.... +-
U Rl R2 Rn
Los valores de Rl , Rz, ... , Rn son 50. 100, l SU, .... 50n ohmios respectivamcnre.
con Ull error porcentual máximo de 0.8% ~Ias mediciones. Esti me el error
porcentual máximo en el cálculo del va lor de 1L",
"
Solución
Las derivadas parciales de U con respecto a Ri son
aR RZ
aR( =: R~' i =: 1,2, ... , n

De donde resu Ita

RZ RZ RZ
elR=-dR +-elR
RZ +···+-dR
R12 1
'2
Z R2
n
n

dR
Como E.P.(RJ == - ( x (lOO) =: 0,8%, i =: 1,2, ... , n. enronces el error
Ri
porcentual de R es aproximadamente

R dR~ F? dRz R dR ll
E.p.eR) == - . - x 100 + - . - x 100 + ... + - . - x 100
R¡ Rl Rz Rz Rn Rn
R R R
== Rl (0,8%) + Rz (0,8%) + ... + Rn (0,8%) =: 0,8%
Ejemplo 46.- Consideremos el triánguln
isósceles T de la figura 3.11. ¿Cuál e,
aproximadamente el cambio que ocurre en el
éirea de T si las longitudes de sus lados iguab
:llIInentan en una pulgada ~ el úngulo en el
vértice se incrementa en O,O"¡ radianes'?
Solución
Si x es la mcdida de la longitud de los lados ;-
a el ángulo comprendido entre ellos, entonce,
el úrca del triángulo es
1 ,
A = -x~
2 .sen Ca)
Luego,
aA aA J
dA = -ax dx + -aa da = "( sen (a)dx + -x
2
2
cos(a)da
1
Como dx = 12 pies, da = 0,04 radianes; el cambio que ocurre en el área del
triángulo es

Ejemplo 47.- Cuando se calcula el área de ~1 trapecio isósceles se obtiene


256 cm 2 con un error porcentual de 4%. Si se"'Stbbe que al medir las longitudes de
<}o

la base menor b, base mayor B y de la altura h , los errores de medición fueron


iguales, halle los errores porcentuales al medir b, B Y h respectivamente, cuando
b = 12 cm, B = 20 em y h = 16 cm.
So lución
El á.rea deJ Jrapecio .isDsreJes es

A=---
eb+ B)h
2
De donde resulta
(b + B)
dA = -aA
ab
aA
aB
aA
ah
h
2
h
db + - dB + - dh = - db + - dB + - - dh
2 2 I

Como db = dB = dh, h = 16, b = 12 Y B = 20, el error aproximado en la


medida del área es
dA = 8db + 8db + 16db = 32db C*)
Puesto que el área medida es A = 256 cm 2 y su error porcentual 4%, se tiene
dA 4
E. P. CA) = A x 100 = 4 ~ dA = 100 A = 10,24
Luego, de (*) se tiene
10,24
1024I
= 32db (:::::) db = -32- = 032
'
= dB = dh
Por tanto, los errores porcentuales al medir b, B Y h son
db (0,32)
E.P(b) == b(100) = U (100) = 2,67%

dB (0,32)
f. P(B) == B (100) = 20 (100) = 1.6%

(0,32)
E. P(h) == hdh (100) = 16 (lOO) = 2%

Eje mplo ·t8.- La utilidad men sual en nucvos soles dc una empresa que
comcrcializa un solo producto es dada por
1 1

U(x; y) = 50 (x' + 2xy),

Jonde x reprcsenta el número de unidades vendidas en Lima e y el nlJmcro dc


unidades vcndidas en Arequipa. Si en la actualidad la empresa vende 200
unidades cn Lima y 300 unidades en Arequipa, estime el cambio aproximado en
la utilidad de la cm presa si las y.cntas en Lima disminuycn cn 1%, mientras quc
{~,~
en Arequipa aumcntan en 2%.
Solución
Como las ventas de la empresa di sminuyen en Lima el1 un ! ~;, Y aUlllelllan en
t\rcq uipa en un 2%, entonces sc ticne
dx = Óx = -200(0,01) = -2 \' dy ~ = 300(0,02) = (,
.~,
La dit"crcncial total dc la utilidad cs
1 x
dU(x;y) = UxCx;y)dx + Uy(x ;y)dy = 25 (x + y)dx + 25 dy

Para x = 200 YY = 300, se tiene


dU(200; 300) = 20 dx + 8dy = 20( -2) + 8(6) = 8
Luego, ÓU(200; 300) == dU(200; 300) = 8
Por tanto. la utilidad mcnsual de la cmprcsa aumenta aproximadamcnte en S. . 8
111cnsua1.

Ejemplo 49.- Se fabrica un recipicnte sin tapa que tienc la ...


lorma de un tronco de cono circular con base una semiesfera.
1,11 como se muestra en la figura adjunta. Las dimensiones del I
20

1
I cC lpicntc son

U = 18cm y r = 12 cm
si al recipientc sc dcsca bai'lar con una capa de plata Je 0.0 I
l lll de espesor, estimc el volumcn aproximado dc plata que se
Ill'ces ita para bariú la superficie cxterior.
Solución
El volumen del tronco de cono circular recto de radio mayor R. radio menor r y
altura h es
rrh 20rr 2
Vrc = - (r 2 + rR + R2) = - (r + rR + R2)
3 3
Y de la esfera de radio r es
4 3
VE = 3 rrr
Luego, el volumen del recipienre es
20rr 2rr
V=-(r 2 +rR+R 2 )+-r 3
3 3
La diferencial total del volumen V es
av av rr rr
dV = -;-
ur
dr + ; ¡ dR = - (40r + 61'2 + 20R)dr + - (20r + 40R)dR
uR 3 3
Para R = 18, r = 12, dR = óR = 0,01 Y dr = /).r = 0,01; se tiene
rr rr
dV = - (480 + 864 + 360)(0,01) + -
(240 + 720)(0,01) = 8,88rr
3 \,' 3
Por tanto. para bañar la superficie exterior del recipiente se necesita
aproximadamente ÓV == 8,88rr cm 3 de plata.

Ejemplo 50.- Sea [(x; y) = (x 3 + y3)JX 2 +~, ¿es [diferenciable en (O; O)?
Solución .~~

Por el Teorema 7 podemos ver que [ es diferenciable en (O; O) si


a[(x; y) af(x; y) .
[(x; y), a y son contll1uas en (O; O).
x ay
i) Como lim f(x>y) = O = feO; O), concluimos que f es continua en (0;0)
(x;y)->(O;O)

ii) La derivada parcial de [ con respecto a x es


X(X3 + y3)
a[(x;y) 3x 2JX 2 +y2+ ,si(x;y):t:(O;O)
h(x;y)= ax ={ JX 2 +y2
O , si ex; y) = (O; O)
afeO' O) [(h' O) - [(O' O)
pues. ' = lim' , = ¡im h 2 1hl = O
ax h ->O h IHO

Ahora, al utilizar coordenadas polares, se obtiene


lim h(x; y) = liml3r 3 cos 2 e + r 2 cos e (cos 3 e + sen 3 e)] = O = h(O; O)
lX,Y)->lO ;O) r->O

a[(x;y)
Así, h(x; y) = ax es continua en (O; O)
iii) La derivada parcial de f con respecto a y es dada por
Y(X3 + y3)
af(x;y) 3y2"fx 2 +y2+ ,si (x: y) *(0;0)
H (x; y ) = ay =
{
-JX2 + y2
o , s i (x; y) = (O; O)

afeo; O) . feo; k) - feo; O) k 3 -..[k2


donde ay = 11m
k~o k
= I i m - - = Iim k2 1kl = O
k --> O k k-->O

En forma similar, al usar coordenadas polares se tiene


Iim H(x;y) = Iim[3r 3 sen 2 e + r 2 sen e(cos 3 e + sen 3 e)] = O
(x;y)-->(O;O) r-->O

= H(O; O)
af(x;y)
Luego, ay es continua en (O; O)
Por consiguiente, f(x; y) es diferenciable en (O; O).

, EJERCICIOS

1.- Sea f(x; y) = {'JXY + x:ty' .si ex; y) * (O; O)


O, si (x; y) == (O; O)
¿Es f diferenciable en los puntos (O; O), (e~\ (1; 1)? Justifique R. no, no, si
.~~

2. - Sea la función

f ( x;y ) ==
ex + eY +
x
2 xy
+Y
2' si (x; y) *' (O; O)
{
2 , s i (x; y) == (O; O)
Analice la diferenciabilidad en (x; y) == (O; O) R. no es diferenciable

3. - Para las siguientes funciones, analice la diferenciabilidad en el punto (0;0)

a)f(x;y )= t' (X2 + y2) cos (

O
"fx 2 +y2
1. ), si (x; y)

, s i (x; y) = (O; O)
*' (O; O)

b)f ( x;y ) ==
X sen ( 2Y 2) ,si (x; y) (0;0)
x +y
*'
{
O , s i (x; y) = (O; O)
4 .- Sea la función f(x; y) == ~,(x; y) E rn¡,2. Determine el conjunto de puntos
donde f no es diferenciable. R. f no es diferenciable en los eje X e Y.
5.- En cada uno de los ejercicios halle d[(x; y) y tJ.[(x; y) para los valores
dados de x, y, tJ.x y tJ.y.

a) [(x;y) = X2 -xy+ y2 ,x = 2,y = -l,tJ.x = -O,Ol,tJ.y = 0,02


rr
b) [(x; y) = senxy + cos(x + y),x = 6'y = O,tJ.x = 2rr,tJ.y = 3rr
rr rr
c) [(x; y) = eXYsen (x + y),x = 4'y = O,tJ.x = -4'tJ.y = 4rr
xy
d)[(x;y)= 2 2,x=3,y=1,tJ.x=-0,1,tJ.y=0,05
x +y
e) [(x; y; z) = X2 - 2y 2 + Z2 - xz
(x; y; z) = (2; -1; 3), (tJ.x; tJ.y; tJ.z) = (0,01; 0,02; 0,03)
f) [(x; y; z) = sen (x + y) - cos(x - z) + sen (y + 2z)

(x;y; z) = G;~; O), (tJ.x; tJ.y; tJ.z) = (¡;i; 2rr)

6.- Halle el valor ap~oximado J&las siguientes cantidades utilizando diferenciales.


a) J(5,02)2 + (11,97)2 b) J(3,02)2 + (1,99)2 + (5,97)2
c) (3.01)2 + (3,98)2 + (6,02)2 + (1,97r1/2 R. 0 ,111427
d) sen 32° cos 59° R.O,273
e) In[(1,1)3 + (2,3)3] -In 9 R.0,43

7.- Un envase de metal que tiene la forma de un cilindro circular recto tiene una
altura interior de 8 pulgadas, un radio interior de 3 pulgadas y un espesor de
0,2 pulgadas. Si el costo del metal que va a ser usado en su construcción
cuesta SI . 10 por pulg 3 , encuentre el cos.ro aproximado del metal que se usará
en la manufactura del envase. R. SI. 132rr

A
8.- La gravedad específica S de un objeto está dada por la fórmula S = A _ W
donde A es el número de libras de peso del objeto en el aire y W es el número
de libras del peso del objeto en el agua. Si el peso de un objeto en el aire es 20
libras con un posible error de 0,0 I libras, y su peso en el agua es 12 libras con
un posible error de 0,02'libras, encuentre el máximo error posible al calcular S
a partir de estas medidas. Tambiéf) halle el máximo error porcentual.
7
R. 1600; 0.18%
';1.- Una compallla va a manufaclUrar 1(JOOO cajas de madera cerradas con
dimensiones 3 pies, 4 pies y 5 pies. El costo de ia madera que va a ser usada es
de 5 soles por pie cuadrado. Si las maquinas que se usan para cortar las piezas
de madera tienen un posible error de O,OS pies en cada dimensión, hallc
aproximadamcnte usando la diferencial total. el maximo error posible en la
estimación del costo de la madera. R. 120000 soles

10.- Si cada individuo y cada corporación en la economia de un pais gasta una


proporcioll x de cada sol recibido, entonces por el principio muitiplicador
v
(keynes), la cantidad de dinero generada por la infusión de y soles es --'-
l-x
Si y se incrementa de I a 1,03 millones y x disminuye de 0,6 a 0,58, ¿Cual es
el cambio aproximado en el monto de dinero generado'), ¿aumenta °
disminuye?

11.- /\1 medir un triángulo se encuentra que los lados


tienen longitudes de 50 1'.J.J)g. Y 70 pulg. y el
angula comprendido entre ellos es de 30°, Si
existen errores posibles de 1/2 % en la medida de
los lados y I /'2 grado en la del angulo. Halle el
máximo error aproximado en la medida d~rea .
R. '2,5%
<p~

kT
1'2.- La ecuación P = ti ' donde k constante, expresa la presión P de un gas
encerrado, de volumen V y temperatura T. Calcule aproximadamente el
porcentaje de error maximo que se comete en P cuando el error en la medida
de T y V es de 1,4% Y 0,9% respectivamente ..
R. Porcentaje de error nuiximo de P es 0,5%
13.- Se desea embalar un televisor cuyas dimensiones son: 55 cm de largo, 4U cm
de ancho y 80 cm de altura. con un material homogéneo cuyo peso es de 11'20
gramos por cm 3 . Si el grosor del embalaje lateral debe ser de 5 cm mientras
que el de la base y la parte superior de '2.5 cm cada uno, use diferenciales y
calcule aproximadamente ei peso de la envoltura.
14.- Cuando un medicamento se ingiere, el tiempo T en el que hay mayor
cantidad del producto en la sangre se puede calcular en términos de la
semi vida "x" del medicamento en el estómago, y la semivida "y" en la sangre.
Para fármacos comunes T esta dado por
xy(ln x - In y)
T=------
(x - y) In 2
Para un medicamento en particular x = 30 mlll e y = 1 hora. ¿Cuál es el
máximo error porcentual en el cálculo de T si el error máximo en la
estimación de las semi vidas es de IO%? R. 10%

15.- El radio de la base y la altura de un cono circular recto miden 6 cm y 8 cm


respectivamente, con un posible error en la medición de O, l cm en cada
dimensión. Utilice diferenciales para estimar el error má>..nno posible en el
cálculo del área de la superficie lateral.
(Sug. Área lateral= rrr~r2 + h2 ) R. 1,84rr cm 2

16.- Se tiene un cilindro circular recto metálico cuyas dimensiones son 2 metros
de altura y I metro de radio en la base. Se desea pintar exteriormente con una
capa de pintura de 0,002 m de espesor tanto en la parte lateral, como en las
bases. Use diferenciales para estimar la cantidad de pintura que será necesario.
"""'" R. 0,012rr m 3
17.- La producción mensual de la fábrica Sajita S.A. es
P(x; y) = 60Xl/3 y l/2 unidades
donde x representa el capital invertido en miles de soles e y el trabajo medido
en horas de trabajo. En la actualidad, el c~al mensual invertido es de 8000
soles y se emplean cada mes 900 horas de tral:mjo.
a) Calcule la producción actual
b) Halle el aumento aproximado en la producción de la empresa que resultó de
aumentar el capital en 2000 soles y la fuerza de trabajo en 2 horas.
R. a) 3600 unidades b) aumenta en 154 unidades.

18.- Se quiere construir una pieza mecánica de acero que tiene la forma de un
paralelepípedo recto de base cuadrada. Si al medir el lado del cuadrado de la
base y al calcular el volumen de dicha pieza mecánica se han cometido errores
de 1% Y 4% respectivamente, ¿cuál es el error porcentual cometido al medir la
altura del paralelepípedo? R. 2%

19.- El ancho, el largo y la profundidad de un tanque que tiene la forma de un


paralelepipedo rectangular han sido medidos con errores de 2%, 1,5% y 2,5%
respectivamente. Estime el error porcentual máximo al calcular el volumen de
dicho tanque. R. 6%
t6 REGLAS DE LA CADENA PARA UNA FUNCIÓN DE VARIAS
VARIABLES

Recordemos que la regla de la cadena para funciones diferenciables de una


variable establece que si y::= [(u) es una función derivable de u y u ::= g(x) es
una función derivable de x, entonces la derivada de la función compuesta
y ::= [(g(x)) es dada por

dy dy du
dx du :dx
Fn el siguiente Teorema consideraremos la regla de la cadena para funciones de
dos variables. donde cada una de las variables independientes es también función
de dos o más variables independientes.

Teorema 8.- Sea z ::= [(x; y) una función diferenciable de x e y. Si x ::= ter; s)
y y ::= [(x; y) son tales que las derivadas parciales de primer orden
()x ax ay ay . . '. . .
-) '-a '-a y -a eXisten; ent~ces la funclOn [ es de forma mdlrecta
(r s r s
función de r y s, y se tiene
az az ax az ay
ar::= ax' ar + ay' ar (1)

az az ax az ay
-::=- -+-- (2)
as ax' as ay' as

derivadas parciales dadas en las igualdades (1) Y(2).

0= 0= OX 0= ay
-=--+--
or OX . or ay' or

0= 0= OX 0= ay
-=-.-+-.-
Variables os OX os ay or
intermedias Variables independientes
Teorema 9.- (Regla de la cadena general), Sea z = ¡(Xl; .. ,; x n ) una función
diferenciable de n variables. donde cada Xi es una función diferenciable de
variables tI' .. , tm' esto es,
ax
Xi = F;(t l , .. " tm) y -at, í = 1,2, .. " n y j = ,1,2, ,.. , m existen,
tj
Entonces la función z = ¡(Xl; .. ,; x n ) es de forma indirecta función de tI'"'' t m )-
se tiene
az az aX I az aX2 az aX n
-=-.-+-.-+"'+-,-
at l aX I at l aX2 at l aX n at l
az az aX I az aX2 az aX n
-=-,-+-.-+"'+-,-
at 2 aX I at 2 aX2 at 2 aX n at 2

Corolario.- Sea z = [(Xl' .. " x n ) una función diferenciable de n variables, Si


cada Xi (í = 1,2, .. " n) es una función derivable de una única variable t. entonces
z es función de e y la derivada total de z con respecto a t es

dz az dX I az dX2 az ~xn
- = - . - + - . - + '" + - , - 4 0 ; -
dt aX I dt aX2 dt aXn dt

Ejemplo 5J,- CaJcJJJe< me!Jjante la regJE De .lE cEDena .las Der,;vaDas parc.ia,les ql.'e
se indican a continuación:
az az
a) ar y as' siendo z = eX In(y) ,X = S + 2r, y = rs

aw aw r
b) ar y fu,siendow=zy2+x3+y,x=r2+s, y=-r+s, z=~

dz
c) dt,siendoz = xy2 - y, X = e- t , y = sen t

Solución
a) La regla de la cadena para este caso es
az az ax az ay
=- - + - -
ar ax' ar ay' ar
az az ax az ay
-=- -+--
as ax' as ay' as

Las derivadas parciales de z con respecto a x e y son


az'
- = eXlny
ax '

y las derivadas parciales de x e y con respecto a r y s son


ax ax ay ay
ar = 2, -= 1 -=s y -=r
as ' ar as
Luego, al reemplazar cada una de estas derivadas parciales en la regla de la
cadena. se obtiene

OZ eX e S+ 2r
- = (eX Iny). 2 + - . S = 2e s+2r .ln(rs) +--
ar y r

b) La regla de la cadena para este caso es

aw aw ax aw a~_ i}w az
- = - - + - -+----
ar ax . ar ay' ar oz' ar

aw aw ax aw ay aw az
-
as
=ax
-.-
as
+ay-.-
as
+az-. as
-

Las derivadas parciales de w con respecto~, y y z son


<}

aw aw aw
- = 3x 2, - = 2zy + 1, _ = y2
ax ay az

y las derivadas parciales de x. y y z con respecto a r y s son


ax ax ay aya? 1 az r
ar = 2r, as = 1, ar = -1, as = 1. ar =
s' as S2

Luego, al reemplazar cada una de estas derivadas parciales en la regla de la


cadena, resulta
aw 1
- = (3x 2). 2r + (2zy + 1). (-1) + y2._
ar s
3r 2
= 6r 5 + 12r 3 s + 6rs 2 + - - 4r + s - 1
s
~: = (3x 2 )(1) + (2zy + 1)(1) + y2 (- ;2)
1'3
= 31'4 + 61'2 S + 3s 2 - -
S2
+r +1
c) La regla de la cadena para la derivada total es
/X_l
dz az dx az dy ::/
-=--+--
dt ax· dt . ay· dt ------- ,. - 1

Las derivadas parciales de z con respecto a x e y son


az ~ az
ax = y', ay = 2xy - 1

y las derivadas ordinarias de x e y con respecto a t son


dx -t dy
dt = -e , di = cos t
Luego, al reemplazar cada una de estas derivadas en la regla de la cadena.
resulta
dz
dt = y2. (-e- t ) + (2xy - 1). cos t
'>~.
= -e- t sen 2 t + 2e-" t sén t cos t - cos t

Ejemplo 52.- Muestre que la sustitución x = eS ,y = e t convierte la ecuación

X2 G:~) + y2 (~:~) + x (~~) + y G~~ O


a2 u a2 u
en la ecuación as 2 + at 2 = O, donde u = [(x; y)
Solución
La regla de la cadena para la función u = [ex; y) es
au au ax au ay au
-=-
as
- + - -=e s -
ax· as ay· as . ax
au au ax au ay au
-=--+--=es -
at ax· at ay· at . ay

Al derivar cada una de estas funciones con respecto a s y t, se tiene

(Por regla del producto)


2
a u _ a [ t aU] _ t aU t a (au)
at2 - at e . ay - e . ay + e . at ay (Por regla del producto)
2 2
_ au
t t [a u ax a u ay]
- e . ay + e axay' at + ay2 . at

Ejemplo 53.- Una lancha se dirige hacia el sur con una velocidad de 8 km/h y otra
hacia el este a 12 km/h. A las 16 horas, la segunda lancha pasa por el punto donde
la primera estuvo hace tres hori'~.
a) ¿Cómo varía la distancia entre ellas a las 15 horas?
b ) ¿Cómo varía la distancia entre ellas a las 17 horas?
Solución
a) A las 15 horas, las lanchas se encuentran e~ puntos A y B (Fig. 3.13), cuya
distancia entre ellas es dada por "'" ..- _ f
z /'
z = J X2 + y2 -____... )' _ 1
También, a las 15 horas la primera lancha se encuentra a y = 16 km del punto
de cruce y la segunda a x = 12 km del punto de cruce.
Al aplicar la regla de la cadena, resulta
dz az dx az dy x dx y dy
-=- -+- -= 2 -+ 2 -
dt ax'dt ay'dt JX +y2'dt JX +y2'dt

dx dy
Luego, para los datos x = 12, Y = 16, dt = -12 Y dt = 8, se tiene

-dz
dt
-20 (-12) + (16)
= (12) -20 (8) = -O ' 8
Por tanto, la distancia entre las lanchas disminuye a una velocidad de 0,8
km/h.
y~
y

A x x

~ ti'
O x X

~= 12kmlh
Y

B Y Z
~=- 12km l h d,· dI
dI ....!.-= 8km l h
dI B
!!.L = &km Ih
dI

Fig 3 14
Fig 3 13
I
b) En forma similar. a las 17 horas lFig. 3.14) x = 12 km. y = 32 km y ia
distancia entre \as \anchas es dada por
z = ~X2 + y2
Luego, la regla de la cadena en este caso es
dz az dx az dy'" x dx y dy
-=- -+- -= -+ -
dt ox'dt ay'dt ~x2+y2'dt Jxz+yz'dt

dx dy
Luego, para los daros x = 12, Y = 32, de = 12 Y di = 8, resulta

dz 12 32~ "
- = (12) + CS) = 11,7
dt .../12 + 32 2
2 2
.../12 + 322
Por consiguiente. a las 17 horas, la distancia entre las lanchas aumenta a una
velocidad de 11.7 km /h.

Ejemplo 54.- Una piscina tiene 22 pies de ancho.


56 pies de largo. 5 pies de profundidad en un 56
eXTremo y 12 pies en el otro extremo, siendo el
fondo un plano inclinado, Si la piscina esta 5
Ilenandose con un caudal de 20 pies 3 jseg, ¿a
que velocidad se esta elevando el nivel de agua
cuando dicho nivel es de 7 pies en el extremo mas
profundo?
Fig . 3.15
Solución
Dc la figura 3.15. el volumen de agua cn la piscina viene dada por
1
V =Z(xy)(22) = 11xy
La regla de la cadena en este caso es
dV av dx av dy dx dy
di = ax'di + ay' dt = llYTt + llxTt (1)
Por semejanza de triángulos (Fig. 3.15), se tiene
y x
=
7 56
Luego, al derivar con respecto a t ambos lados de la igualdad, se obtiene
1 dy 1 dx dx dy
7·Tt = 56' dt <=> dt = 8· Tt ... (2)

Así. al reemplazar (2) en (1) resulta


dV dy dy dy
di = 88YTt + 11(8Y)Tt = 176YTt
dV
Al sustituir y = 7 Y di = 20 pies 3 / seg, se obtiene

dy dX
20 = 176(7) - <=> ~ 001623
dt dt .
Por tanto, cuando la profundidad del agua es 7 pies, el nivel del agua aumenta a
una velocidad de 0,01623 pies/seg.

Ejemplo 55.- En un instante dado, la longitud- ( un cateto de un triángulo es 20


~.

pies y está aumentando a razón de 2 pies/seg. y la longitud del otro cateto es 24


pies y está disminuyendo a razón de 4 pies/seg. Encuentre la rapidez de cambio de
la medida del ángulo agudo opuesto al cateto de longitud de 24 pies en el instante
dado.
Solución
e
Scan x, y y las medidas de los catetos y el ángulo del triángulo rectángulo
(F ig. 3.16). Luego, se tiene

tan e = ~ <=> e = arctan (~)


x . x
I.a regla de la cadena para la variable e es
x
de ae dx ae dy
-=- -+--
dt ax . dt ay' dt
y dx x dy y
--...,- --+ X2 + y2 . -dt
X2 + y2 . dt
Fig . 3.16
dx dy
Al sustituir los datos x = 20, Y = 24, dt = 2 pies / seg y dt = -4 pies / seg
en la regla de la cadena, se obtiene
de 24 20 128
dt = - 20 2 + 24 2 (2) + 202 + 24 2 (-4) =- 976 = -0,131
Luego, el ángulo agudo e disminuye a una razón de 0,131 rad/seg.
Ejemplo 56.- Un filtro cónico de 18 cm de profundidad y 6 cm de radio en la
parte superior, se encuentra llena de una solución. La solución va pasando a un
vaso cilíndrico de 3 cm de radio. Cuando la profundidad de la solución en el filtro
es 12 cm y el radio 4 cm, su nivel está bajando a razón de 2cm/seg y el radio va
decreciendo a razón de 2/3 cm/seg. Halle la rapidez con que está subiendo la
solución en el vaso, para dichas medidas,
Solución
En la figura 3.17 se observa que el volumen de
la solución en el filtro cóni&,o de radio r y
">,
altura h es dado por

r 2h
H = 24--
27
Luego, la derivada total de H con respecto a t es

dH aH dr aH dh 2rh dr r 2 dh
-=- -+- -=-- -----
dt ar . dt ah . dt 27 . dt 27' dt
dh dr 2
Al sustituir los datos r = 4, h = 12, -dt = -2 cm/ seg y -
dt
= --cm
3
iseg
en la derivada total, resulta

dH = _ 2(4)(12)(_~)_~(_2) = 32
dt 27 3 27 9
Por tanto, la solución en el vaso cilíndrico sube a una velocidad de 32/9 cm/seg.

Ejemplo 57.- Un corredor va por una pista circular de 40 metros de radio a razón
de 8 m/seg. En el centro de ésta hay una luz. la sombra del corredor se proyecta
sobre un muro recto tangente a la pista en el punto de partida. ¿Con qué rapidez se
mueve la sombra cuando lleva recorrido 1/8 dc la pista?
Solución
El arco S = AB recorrida por el atleta es dado por
S = rO
Como la sombra del corredor se proyecra sobre la
recta tangente LT , entonces se tiene (Fig. 3. J 8)
tan e = ~
r
Luego, la distancia recorrida por la sombra de J
corredor (AT = y) viene dada por
y = rtane (r = 40m fijo)
La derivada total de las variables y y S COIl
respecto '21\ \:,cnipo t son

dS as dr as de de
-=-.-+-.-=r-
dt ar dt ae dt dt
De estas dos ecuaciones, resulta

1 2rr rr dS
Al sustituir los datos e = "8 (longitud de la pista) = 8 = ¡ y dt=8m j seg

en la derivada total ~~, se obtiene


dy
- = 2(8) = 16
dt
Por consiguiente, la sombra del corredor se mueve sobre la recta tangente a una
rapidez de 16 miseg.

Ejemplo 58.- Sea [: D e ~2 -> ~ una función diferenciable. tal que

[(l8; O) = 4 Y Dd(l8; O) = Dzf(l8; O) = 3


Si H (x; y; z) = [(x 2 - y2 + Z2; y2 - Z2 + x 2 ), halle la ecuación del plano
tangente a la superficie S: H(x; y; z) = O en el punto Po(3; -4; 5)

Solución
Al considerar u = X2 - y2 + Z2, V = y2 - Z2 + X2 y H(x; y; z) = f(u; v) = O,
se tiene
8H(x; y; z) 8f(u; v) 8u 8f(u; v) 8v
8x = 8u . 8x + 8v . 8x

= D1 f(u;v).2x + D2f(u;v).2x

8H(x;y;z) 8f(u;v) 8u 8f(u;v) 8v


--'---- = - + -
8y 8 u ' 8y 8 v ' 8y

= Dd(u; v). (-2y) + Dzf(u; v). (2y)


8H(x; y; z) 8f(u; v) 8u 8f(u; v) 8v
--8'--z-- = 8u . -8z + 8v . 8z

= Dd(u; v). (2z) + Dzf(u; v). (-2z)


Al evaluar estas derivadas parciales en el punto Po(3; -4; S), se tiene
8H(3' -4' S) é1ii(3' -4' S) 8H(3' -4' S)
, , = 36 .~, , =OY " =O
8x '8y 8z

Por tanto. la ecuación del plano tangente a la superficie S en el punto Po(3; -4; S)
es
PT : 36(x - 3) + O(y + 4) + O(z - S)~ PT:x = 3

Ejemplo 59.- Sea f una función diferenciable, tal que

f(2; 2) = 2, Dd(2; 2) = -2 Y Dzf(2; 2) =4

Solución
i) g(2) = f(2;J(2; f(2; 2)) = f(2; f(2; 2)) = f(2; 2) = 2
ii) Al considerar v(x) = f(x; x), u(x) = f(x; v(x)) y g(x) = f(x; u(x)), se
tiene
g'(x) = Dd(x; u(x) ).1 + Dzf(x; u(x)). u'(x)

= Dd(x;u(x)) + Dzf(x; u(x))[Dd(x; v(x)) + Dzf(x; v(x)). v'(x)]

= Dd(x; u(x)) + Dzf(x; u(x))Dd(x; v(x)) +


+Dzf(x; u(x))Dzf(x; v(x))[Dd(x; x) + Dzf(x: x)]

Luego, al evaluar en x = 2 resulta


g'(2) = -2 + 4[-2 + 4[-2 + 4]] = 22

Ejemplo 60.- Transfonne la ecuación


. (aW)2
ax + (aw)2
ay = O, al tomar como

variables independientes a r = JX2+ y 2 V


-
e = arctan (~l
X'
Solución
El diagrama de árbol de dependencias se muestra e
cadena para w viene dada por -
aw aw ar aw ae
-=--+--
ax ar .ax ae .ax
aw (X) aw ( y)
= ¡¡;: -;: + 08 - rL

aw aw ar aw ae
-=--+--
ay ar .ay ae .ay
= aw (~) + aw (~)"'l» Fig 3 19

ar r ae r 2
aw aw
Al sumar los cuadrados de las derivadas parciales ax y ay' se obtiene
+ (aw)2 _ (aw)2 + ~ (aw)2 ~
(aw)2
ax ay - ar r ae, 2

Por tanto, la ecuación resultante es


aw)2 1 (aw)2
(a:;: + r 2 ae = O

3.7 DERIVACiÓN IMPLÍCITA

Se dice que la ecuación


F(x;y;z) =0 (*)
de fine en forma implícita a la función f: D e !R1. 2 Iffi., si al sustituir z = {(x; y)
-1

en la ecuación (* l, ésta se reduce a una identidad. esto es.


F(x;y;f(x;y)) = O,V(x;y) E D
Po r ejemplo, la ecuación
F(x; y; z) = 3x 2+ 4y2 + Z2 - 12 = O
re presenta en forma implícita a las funciones
7. - [(x; y) = V12 - 3X2 - 4y2 Y z = g(x; y) = - JT2 - :~ '1 , ~
dI' la I'unción Implícita 10.- Sea F: D e [RI.ll+1 ~ [RI. una función real
111111 ' 111,1

dI /1I I \<lriables tal que:


1)F(X 1;oo.;X n ;Z) =0 (*)
ii) F tiene derivadas parciales continuas en la vecindad del punto P(x l ; oo.; X ll ; z)
iJF(P)
iii) - - -
iJz
*O
Entonces la ecuación (*) representa a z en forma implícita como función de
Xl; oo.; XI!' esto es, z = {(Xl; oo.; XI!) y para P(x 1 ; oo.; Xn ; z) E D. se tiene

iJF(x 1 ; oo. ; Xll ; Z)


iJ{(X 1 ; ... ; Xll ) iJX¡
-(-:-----=----:-)-' i = 1,2, oo. , n
iJX¡ F Xl; oo.; X,,; z
iJz

iJz az
Ejemplo 61.- lIallc iJx y iJy,Siz={(x;y)satisfacelaecuación
xy2 + yz2 + Z3 + x 3 -t:z,- 4= O
Solución '2.

Sea F(x; y; z) = xy2 + yz2 + Z3 + x 3 - 4 = O


Las derivadas parciales de F con respecto a X, y Y z son
JF " _, aF ,iJF .,
- = y" + 3x·, -a = 2xy + Z", - = 2yz + 3r
iJx y iJz
Luego. al reemplazar cada una de estas deriv'1h8as en la forma de la derivación
implícita. se tiene
iJF
iJz y2;+-3y2
ú:C'
iJx iJF 2yz + 3z 2
iJz
iJF
iJz ay + Z2
2xy
ay - aF 2yz + 3z 2
az

Ejemplo 62.- si u, v son funciones de X e y definidas implícitamente en alguna


región del plano XY por las ecuaciones
U sen v + X2 = O
{ U cos v - y2 = O

iJu iJu ( , iJ 2 v iJ 2 v\
Ibll,,¡:;'=,,--Y--I-lI )'"_-1-, ;,v2_)'-l-, vhv()v()
--- - - ./ ay .. ax ... \ ux 2 .. iJ y 2
Solución
Al derivar en forma implícita ambas ecuaciones con respecto ti X. Sl! til!lll!
au av
ax sen v + u cos v ax + 2x = O
{ -au cos v - u sen v -av = O
ax ax
au av
Luego, por la regla de Cramer las variables ax y ax vienen dadas por

au ¡-~x u cos v ¡ 2xu sen v


-usen v
=
ax ¡sen v u cos v ¡ -u(sen 2v + cos 2v)
= -2xsenv
cosv -u sen v
[sen v -;x[
av cosv 2x cos v
ax -u u
En forma similar, se obtiene

au a~ 2y sen v
ay = 2ycos(v) y ay;::: - u

Las derivadas parciales de segundo orden son:

a2v U[2COsv-2xsenv.*]-~·cosv.*
-=- )¡
ax2 u2 'e,

2u cos v + 8X2 sen v cos v


u2

02V u (2 sen v + 2y cos v.~l- 2y sen v .~

2u sen v - 8y2 sen v cos v


=
u2

Luego, al sustituir estas derivadas parciales en la expresión de E, se obtiene


2 2
E = y -au
ay
- x -au + U [y2a- v
ax ax2
+ X2 -a v] + 600 = 600
ay2

Ejemplo 63.- Si ¡ex - z; y - z) = o define en forma implícita a z como funcióll


az az
de x e y, halle ax + ay
Solución
Al considerar a =x - z y b = Y- z, se tiene
F(x;y;z) =f(a;b) = O

Luego, al aplicar la regla de la cadena y la


fórmula de la derivación implícita, se tiene

aF aF aa aF af
=--
ax aa . ax aa aa
aF aF aa aF ab aF af
ay = aa· ay + ab· ay = ab = ab Fig.3.19

aF aF aa aF ab aF aF af af
az = aa . az + ab· az = - aa - ab = - aa - ab

az az
Por consiguiente. ax + ay = 1

Ejemplo 64.- Si z + eX sen (y + z) - 1 = O" fine a z en forma implícita como


'"b
a2 z rr ".
función de x e y. halle -aa
Y x
en el punto (O; -; O)
2
Solución

Sea F(x; y; z) = z + eX sen (y + z) - 1 = O. Luego, al aplicar las fórmulas de


derivación implícita, se tiene

aF aF
az ax eXsen (y + z) az . ay eXcos (y + z)
ax - aF 1 + eX cos(y + z) . ay = - aF = 1 + eX cos(y + z)
az az

E=--
a2 z
ayax
eX[cos(y + z) + eX cos 2 (y + z)] (1 +~) + e 2x
sen 2 (y + z) [1 + ~]
[1 + eX cos(y + z))2
Al evaluar la derivada parcial de segundo orden en el punto Po ( O; i; O) , resulta
EJERCICIOS

1.- En los siguientes ejercicios, obtenga las derivadas parciales indicadas al usar la
regla de la cadena.
ou ou
a) u = (yz)X , x = e s + t , y = S2 + 3st , z = sent 'os'
- -ot
dz
b) z = In(x 2 + y2) + Jx 2 + y2, X = e t cos t, Y = e t sen t, dt
dz
R. dt = 2 + el
dz
c) z = xe xy , x =t- 3, Y = e t - 3 , -dt
d) u = 2x 2 - yz + xz 2, X = 2 sen t, y = t2- t + 1, z = 3e- t ,
du
dt en t =O R. 24
ou ou
e)u = ares en (3x + y), x = r 2e X
, y = sen(rs); . or' as

f)u = cosh (-x


Y) , x = 2r
~
L
s, Y = 6se T ;
ou ou
'-a
, s
x - Y ou au au
g) u =
l+x +y
2 2' X = r + 3s - t, Y =r - 2s + 3t; o ' as' ot
r
au ou au
h) u = xe-y, x = arctan(rst), Y = cos(3rs + Sst); or' os' at

2.- En los siguientes ejercicios, suponga que w es una función de todas las otras
variables. Halle las derivadas parciales indicadas en cada caso.
ow aw
a) 3X2 + 2y 2 + 6w 2 - X + Y = 12 - -
, ox' oy
aw aw
b,. X2 - 2xy + 2xw + 3y2 + w 3 = 71 - , -ox' -ay
~ aw aw
2 2
c) x"y - x w - 2 xy 2 - yw + w3 = 7, ax' ay

3.- Suponga que {(x; y) = e XY y que g, h son funciones tales que g(2) =:~,
g'(2) = 4, h(2) = 5. h'(2) = 6; Halle F'(2) si: F(t) = {(g(t); he!)~
R.3fle 1 "
4.- Si ter; s) es diferenciable en (0;0) y Dd(O; O) = 2. Dzf(O; O) = 3 Y cJ>(u; v)
es diferenciable en (O; O). cJ>(0; O) = O. cJ>,,(0; O) = 7. cJ>u(O; O) = 9.
Sea 9(U; v) = [(cJ>(u; v); u). Calcule 9,,(0; O) y 9u(0; O)
R. 9,,(0; O) = 17. 9u(0; O) = 18

au uDd(xu; y)
5.- Dado u = [(xu; y) demuestre que: ax =
1 - xDd(xu; y)

2 z z a 2
a 2

6.- Si z = cJ>(y + ax) + ljJ(y - ax), demuestre a ay2 - ax2 =O

7.- Si z = [(x; y). x =r cos e.y == r sen e, demuestre que

{rJ'l.,)2 + 1, {íJ'l.,)2 = (íJZ,)2 + fdz,)2


\ar r~\ae \ax \ay

8. - Si u = e x - at cos(x - at) '1-'


v
'·"'n~b.e que - == a 2 -ax2
a2 u a2 u
at2

9.- Dado z = u(x; y)e ax +bY , donde u(x; y) es una función de x e y tal que
a2 u
-a a == O. (a. b constantes) . l lalle los val~ de a y b que hacen que la
xY .
%
a 2z az az
expresión axoy - ax - ay +Z = O R. a = b == 1

~ aw aw
10.- Si w = [(x'- - y2; y2 - X2). pruebe que y - + x- = O
ax ay

11.- Una función u es definida por una ecuación de la forma u = xy [

au au
e ~y)
Pruebe que u satisface la ecuación X2 ax - y2 ay = G(x; y)u y halle
G(x; y). R. G(x; y) = x - y

aF\ 2 .a F 2
12.- Una cierta función F(x;y) es tal que (ax) + (ay) = 2. 'If(x;y)

1
El cambio de variable x = uv. y = "2 (u 2 - v 2 ) transforma la función
F(x; y) en una función de u y v. Determine los valores de las constantes a y b
aF 2 aF 2 1 1
tales que a (-) - b ( - )
au av = u2 + v 2 R. a = -.
2
b = --2

13.- Un lado de un rectángulo de x = 20 m, aumenta con una velocidad de 5


miseg, el otro lado de y = 30 m, disminuye con una velocidad de 4 m/seg.
¿Con qué velocidad variará el perímetro y el área de dicho rectángulo?
R. Perímetro 2 m/seg, área 70 m 2 / seg

14.- El radio de una esfera disminuye a razón de 2 cm/seg y el radio de la base de


un cono recto, inscrito en dicha esfera, aumenta a razón de I cm/seg. Calcule
la rapidez con que varía el volumen del cono cuando el radio de la esfera es
dV
de 10 cm y radio de la base del cono 6 cm. R. dt = 9rr

15.- Una cantidad de gas obedece a la ley de un gas ideal PV = 12T, Y el gas está
en un recipiente que es calentado a una rapidez de 3° por seg. Si en el instante
cuando la temperatura ;} 3t).00, la presión es 6 lib /pulg 2 y está
2
disminuyendo a la rapidez de O.llib /pulg por segundo, halle la rapidez de
cambio del volumen en ese instante. R. Crece a razón de 16 puig 3 / seg
Nota: P es la presión, V el volumen, T la temperatura.

16.- En un instante t, medido en minutos, una c'funche sobre el plano XY está en


el punto (x; y). donde las distancias se miden en pies. La temperatura en
(x; y) es z = T(x; y) = e- x - 2y grados. Cuando la chinche está en el punto
(0;0) se mueve hacia el este a una velocidad de 2 pies/m in . y hacia el norte a 3
pies/m in. Desde el punto de vista de la chinche, ¿con qué rapidez está
cambiando la temperatura del suelo? R. -8°C/min .

17.- Un depósito en forma de un cono invertido, tiene una altura de \O m y una


base de 10m €le diámetro. Si el depósito está llenándose de agua a razón de
2 m 3 / seg, ¿a qué velocidad se está elevando el nivel de agua cuando el nivel
se encuentra a 3 m de la parte superior del depósito?
8
R. - 9 m/seg
4 rr

18.- Uno de los lados de un paralelogramo está aumentando a razón de 10 cmiseg.


y uno de los adyacentes está disminuyendo a razón de 5 cm/seg, mientras que
el ángulo comprendido está aumentando a razón de 2°/ seg. Detertnine cómo
y con qué rapidez está variando el área del paralelogramo en el momento en
que tales lados miden 2,40 111 Y 1.50 m respectivamente. si el ángulo
comprendido es 60 0

19.- Una piscina tiene 30 pies de ancho. 40 pies de largo, 3 pies de profundidad en
un extremo y 8 pies en el otro extremo, siendo el fondo un plano inclinado. Si
la piscina está llenándose con un caudal de 40 pies 3 /seg, ¿a qué velocidad
se está elevando el nivel de agua cuando dicho nivel es de 8 pies en el extremo
más profundo'? R. 1/30 pies/seg

20.- El aulO A se desplaza hacia el norle por una carretera y el auto S viaja hacia
el oes\e por otra carretera. cada uno se aproxima al cruce de estas carreteras.
En cierto momento. el auto A está a 1.5 km del cruce y viaja a 90 km /h.
mientras que el auto S está a 2.5 km del cruce y viaja a 60 km,h ¡.Cuál es la
razón de cambio de la distancia entre los autos en ese momell\o')

21.- El radio de un cilindro cift¡JIIar se incremell\a a razón de (j centímetros por


minuto. y la altura decrecc a raZón de 4 centímetros por minuto. ¿Cuál es la
velocidad con la que cambian el volumen y el área cuando el radio mide 12
cm y la altura 36 cm?
dV ., ~ 2
R. de = 4608rr cm min; dI:: 624rr cm / min
O
/

-!:y

22.- En un \riángulo ABC considere los lados AB y AC y el ángulo fJ que ellos


forman. Suponga que el lado AS aumenta a 1/16 cm /m in. el lado AC
disminuye a 1/ 16 cm/min y que el ángulo!) aumema a 0.02 rad 'min.
De\ermine la velocidad con la que varía el área del triángulo cuando AS midc
3 cm, AC mide 4 cm y el ángulo e mide rr /4 rad.
dS 121-
R. - = --.y2 cm 2 / min
dt 1600

23.- El trigo que cae del orificio del fondo de un gran depósito va formando. sobre
el terreno, un montón en forma de un cono circular reclO. La altura dei cono
mide 5 metros y aumenta a razón de 50 cm en cada minuto. El radio de la base
mide 3 metros y aumenta a razón de 50 cm cada minuto. Halle la variación del
volumen que se experimenta en la unidad de tiempo.
13rr
R. 2 m 3 /min
24.- La utilidad "U" de una empresa que fabrica y vende UIl IIl1i~1l Jlllldlll ' lll
depende del precio de venta "p" (en doláres) de dicho producto y dvl g"~lll tll
publicidad "g" (en doláres) para promocionar el producto.
L .. l' I . bl .
a ecuaclOn que re aClOna as vana es antenores es
U p2
=2 - pg
100
.cl
+ 1000
Por otro lado "'p" y "'g" dependen del número "x" de unidades producidas
mediante las reglas de correspondencia p = h(x) y 9 = H(x).
Adicionalmente, se sabe que h'(lOOO) = 1; h(lOOO) = 40; H'(lOOO) = 2
Y H(lOOO) = 5000.
dU
Calcule e interprete dx evaluado en x = 1000.
R. La utilidad de la empresa aumenta a una razón de $ 9.2 por unidad
producida.

25.- La altura de un triángulo disminuye a razón de 2cm /seg, mientras que el arca
del mismo disminuye a razón de 3 cm 2 /seg ¿A que velocidad cambia la base
del triángulo cuando la altJt-a mide 20 cm y el area 180 cm 2 ? R. 1.5 clll/seg

26.- Si la ecuación [(x 2 - y2 + Z2; y2 - X2 + Z2) = O define a z implícitamente


1 8z 1 8z
como función de x e y, halle w = -.- + -.- R. O
x 8x ay

27.- Sea la función z = [(x; y) dada por la ecuación


sen (x + z) + sen ex + y) = 1
¿Para qué valor de la constante a en el punto (rr; rr /2; rr) se verifica la
2 2
a z a z (az az
ecuación x 8X2 + Z 8y2 = a ay - 8J? R. a = 2rr

EJERCICIOS DE DISCUSiÓN
Verdadero o Falso
1.- Si z = f(x; y) es una función con derivadas parciales continuas hasta el 2do.
orden, y además x = 1 + 2t, Y = 3 - t entonces.

éz a 2z a 2z 8 2z
-= 4 - - 4 - - + -
dt 2 ax2 8xay ay2
2.- Considere que la función z = f(x; y) y las funciones DIf(x; y), Dzf(x; y),
D1Z f(x; y) Y D2 If(x; y) son continuas en una región D e 1Fi 2
y además satisface Dl1 f(x; y) = D22 f(x; y)

a2 z
Si x = r + s, y = r - s, entonces asar = -2

YZ
3.- Si f(x;y;z) =
J
xz
e t2 dt,entonces I1f(4;2;0) = (0;0;-2)
';.::f'

¿ 2 2 au au au
4.- Si u = 1J(x +y +z ), entonces yz ax - 2xz ay + xy az = 1

5.- El ángulo que forman en sLi" "intersección la esfera X2 + y2 + Z2 = 1 Y el


plano y = x es igual a rr /3.

6.- Si el plano dado por 3(x - 1) + S(y + 1) + 7(z - 2) = O es tangente a la


superficie F(x;y;z)=O en (1;-1;2), entonces D}F(1;-1;2) =3.
D2 F(1; -1; 2) = 5 Y DJ(1; -1; 2) = 7

R. VFVFFF

You might also like